foresthillshs.org  · web view“the first object that saluted my eyes when i arrived on the coast...

149
AP _____________________ _____ Sample AP World History 2017 Multiple Choice Questions _____________________ _____

Upload: lamduong

Post on 17-Feb-2019

212 views

Category:

Documents


0 download

TRANSCRIPT

AP__________________________

Sample AP World History

2017 Multiple Choice Questions

__________________________

WORLD HISTORY

SECTION I, Part A

Time—55 minutes

55 Questions

Directions: Each of the questions or incomplete statements below is followed by four suggested answers or completions. Select the one that is best in each case and then fill in the appropriate letter in the corresponding space

on the answer sheet.

Questions 1 - 3 refer to the excerpt below.

“The first object that saluted my eyes when I arrived on the coast was the sea, and a slave ship…waiting for its cargo. These filled me with astonishment, that was soon converted into terror…I was immediately handled and tossed up to see if I was sound, by some of the crew; and I was now persuaded that I had got into a world of bad spirits, and that they were going to kill me… I was soon put down under the decks, and there I received such a salutation in my nostrils as I had never experienced in my life: so that, with the loathsomeness of the stench, and with my crying together, I became so sick and low that I was not able to eat…I now wished for the last friend, death, to relieve me; but soon, to my grief, two of the white men offered me eatables; and on my refusing to eat, one of them held me…and laid me across, I think, the windlass, and tied my feet, while the other flogged me severely...”

Elaudah Equiano, The Interesting Narrative of the Life of Elaudah Equiano. 1789

1) Equiano's account suggests that cultural encounters between Europeans and Africans in the 1700's was most directly shaped by

(A) Muslim influences

(B) interest in science and technology

(C) Trans-Atlantic Exchanges

(D) reliance on agriculture

2) Equiano's account contributed most directly to which of the following trends?

(A) The growth of the ideology of Social Darwinism

(B) greater political autonomy for Africa

(C) demands for worker's rights

(D) the abolition of international slavery

3) The experience described in the excerpt is an example of which of the following historical developments?

(A) The Industrial Revolution

(B) The Triangle Trade

(C) The development of Conservativism

(D) The influence of Nationalism

Questions 4 - 7 refer to the graph below.

GROWTH OF THE WORLD POPULATION, 500 B.C.- 2025 C.E

WordPress.com

4. Which of the following was a significant cause of the trend from 1800 to 1900 shown in the graph?

(A) Active encouragement of urbanization by the Muslimmerchants

(B) Economic and political difficulties in Africa & Asia

(C) Incentives offered by European companies looking to hire foreignmigrants from colonial territories

(D) The urbanization of Europe during the Industrial Revolution

5. The people represented by the graph from 1999-2025 most typically will live in which of the following regions of the world

(A) Asia

(B) Africa

(C) The Americas

(D) Europe

6. Which of the following was a direct effect of the trend in population growth between 1800-1900 shown on the graph?

(A) An increase in tensions between social classes

(B) A major economic depression

(C) An upsurge in religious sentiment

(D) The development of political elections

7. The main trend shown in the graph was most directly associated with which of the following processes occurring in Europe at the time?

(A) The divergence of European and colonial economies

(B) The beginning of an industrialized economy

(C) The migration of Asians into Europe

(D) The revival of Catholicism

Questions 8 - 10 refer to the excerpt below.

Preamble"Mothers, daughters, sisters [and] representatives of the nation demand to be constituted into a

national assembly. Believing that ignorance, omission, or scorn for the rights of woman are the only causes of public misfortunes and of the corruption of governments, [the women] have resolved to set forth a solemn declaration the natural, inalienable, and sacred rights of woman in order that this declaration, constantly exposed before all members of the society, will ceaselessly remind them of their rights and duties..."

Article I "Woman is born free and lives equal to man in her rights. Social distinctions can be based only on

the common utility."

Olympe de Gouges, Declaration of the Rights of Woman and Citizen. 1791

8. The sentiments conveyed in the excerpt most directly opposed the prevailing ideal in the early eighteenth century that

(A) the ability of women to earn wages was a positive development

(B) women should focus on the home and the domestic sphere

(C) women shouldn't enjoy full and equal rights with men

(D) women should educate their children about the rights and responsibilities of citizenship

9. Which of the following developments in the first half of the twentieth century best represented the continuation of the ideas expressed in the declaration?

(A) The 19th Amendment to the U.S. Constitution, granting women the right to vote

(B) Women’s support for WWI

(C) Support for outlawing the production and sale of alcohol

(D) A movement focused on religious revivals and personal conversion

10. Many female supporters of the French Revolution in 1789 broke ranks with which group by 1800's?

(A) The Conservatives

(B) Radical supporters of the French Revolution

(C) Supporters of Napoleon

(D) The Catholic Church

Questions 11 - 13 refer to the excerpts below.

"... Stories have been related to me, which the hearer can scarcely credit, as to the terror of the Tatars (Mongols), which God Almighty cast into men's hearts; so that it is said that a single one of them would enter a village or a quarter wherein were many people, and would continue to slay them one after another, none daring to stretch forth his hand against this horseman. And I have heard that one of them took a man captive, but had not with him any weapon wherewith to kill him; and he said to his prisoner, "Lay your head on the ground and do not move," and he did so, and the Tatar went and fetched his sword and slew him therewith.

Ibn al-Atir, 1220-1221, Muslim historian

“Hulagu Khan handed the rich treasures which had been brought...from the Caliph’s court to the ruler of Rayy…for safekeeping, and had them carried to Azerbaijan as was the booty from Asia Minor, Georgia, Armenia, Luristan and the land of the Kurds…..These invaders burned our great libraries, broke our canals and ditches, destroyed our farms, defiled the true Faith by raising temples to Buddha…attempted to destroy our trade with paper money…”

Muslim historian, Rashid Fadl Abi-l’Hair, 1498

11. The excerpts were written in response to the

(A) Mongol's policy of promoting Islam

(B) Arab empires' failure to protect citizens from the Mongol invasions

(C) Arab Dynasties failures to promote cross-cultural trade

(D) The Mongols ability to conquer previously wealthy and powerful empires

12. The ideas about invasion and conquest expressed by both Muslim historians are most consistent with which of the following?

(A) The military and political traditions of central Asian nomads

(B) The belief in Islam

(C) The concept of religious intolerance

(D) The ideas of the Middle Ages

13. The ideas expressed by both Muslim historians best account for which of the following features of the Silk Road during and immediately after the Mongol conquest of Baghdad.

(A) The increase in tension between nomads and settled populations

(B) The rapid expansion of Islam from Spain to India

(C) The inability of any empire to successfully resist Mongol invasions

(D) The spread of disease and the eventual collapse of the Silk

Road

Questions 14 - 17 refer to the excerpts below.

“His Majesty's government view with favour the establishment in Palestine of a national home for the Jewish people, and will use their best endeavours to facilitate the achievement of this object, it being clearly understood that nothing shall be done which may prejudice the civil and religious rights of existing non-Jewish communities in Palestine, or the rights and political status enjoyed by Jews in any other country."

British Foreign Secretary John Balfour, Balfour Declaration, 1917

14. Which of the following was the most immediate result of the declaration?

(A) Arab nationalist became increasingly united over in their political strategies

(B) the rise in concern about the persecution of Jews in Europe

(C) the rise of Zionism promoting Israel as a homeland for the Jews

(D) The British refused to support the idea and focused on Arab nationalism

15. The decision excerpted most directly reflected a growing belief after the First World War that the world's political borders should be divided according to

(A) racial boundaries

(B) social class

(C) national boundaries

(D) fully autonomous and democratic lines

16. Which of the following sets of United Nations decisions demonstrated the strongest continuity with the idea expressed in the excerpt?

(A) U.N. Israel/Palestine Partition Plan of 1947

(B) Camp David Agreement

(C) Oslo Accords

(D) UN Security Council Resolution 242, requiring Israel to withdraw from any recently occupied territories

17. The Balfour Declaration stood in contrast mostly to which of the following other decisions?

(A) Sykes-Picot Agreement, granting Britain control of Mandate Palestine

(B) The creation of the state of Israel

(C) The withdraw of Israeli forces from the Sinai Peninsula

(D) The UN Partition Plan of 1947

Questions 18 - 20 refer to the excerpt below.

"When the Spanish first journeyed there, the indigenous population of the island of Hispaniola stood at some three million; today only two hundred survive. The island of Cuba... is now to all intents and purposes uninhabited;" and two other large, beautiful and fertile islands, Puerto Rico and Jamaica, have been similarly devastated. Not a living soul remains today on any of the islands of the Bahamas, which lie to the north of Hispaniola and Cuba, even though every single one of the sixty or so islands in the group, as well as those known as the Isles of Giants and others in the area, both large and small, is more fertile and more beautiful than the Royal Gardens in Seville and the climate is as healthy as anywhere on earth. The native population, which once numbered some five hundred thousand, was wiped out by forcible expatriation to the island of Hispaniola, a policy adopted by the Spaniards in an endeavour to make up losses among the indigenous population of that island."

Bartolome de las Casas, A Short Account of the Destruction of the West Indies, 1542

18. The ideas expressed in the excerpt differed from the predominant Spanish approach to colonization primarily because de las Casas was

(A) encouraging Spain to start colonizing the Caribbean

(B) highlighting the decline of the natives as part of colonization

(C) promoting new laws that would make it easier to control the natives

(D) arguing that the Americas is more fertile than Europe, making it suitable for colonization

19. The excerpt best reflects an effort by De Las Casas to

(A) overcome opposition to colonization

(B) persuade other priests to join him in the Americas

(C) encourage the improved treatment of the natives

(D) prevent the abuses of colonization from spreading to the Caribbean

20. Which of the following best represents continuity in the years after 1800 with the account that De Las Casasdescribed in the excerpt?

(A) the treatment of slaves as part of the Trans-Atlantic Slave trade

(B) Latin American independence movements

(C) the violence and bloodshed during the Cuban Revolution

(D) Simon Bolivar's struggle to unify South America into a gran Colombia

Questions 21 - 23 refer to the excerpts below.

"After China has established a powerful government, we must not be afraid, as Western people are, that the government will become too strong and that we will be unable to control it. For it is our plan that the political power of the reconstructed state be divided into two parts. One is the power over the government; that great power will be placed entirely in the hands of the people, who will have a full degree of sovereignty and will be able to control directly the affairs of the state."

Sun Yet Sen, The Principle of Democracy, 1924

"Yet if the Western world is still determined to rule mankind by force, then Africans, as a last resort, may have to appeal to (use) force in the effort to achieve freedom . . . We are determined to be free. We want education. We want the right to earn a decent living, the right to express our thoughts and emotions, to adopt and create forms of beauty. We demand for Black Africa autonomy (self-rule) and independence"

Jomo Kenyatta, Challenge to Colonial Powers, 1945

21. The statements of both Sun Yet Sen and Jomo Kenyatta share the same goal of?

(A) using violence as a means of achieving freedom

(B) breaking free from the chains of colonization to establish an independent nation

(C) spreading the ideals of democracy across the entire region

(D) avoiding a military conflict with the imperialists

22. Sun Yet Sen issued the speech primarily in order to

(A) show the state along with the people will influence political affairs

(B) gain support from the Communists

(C) protect nationalists interests in Manchuoko

(D) support the landowning classes of Chinese society

23. Kenyatta's speech best reflects which of the following developments in the decolonization of Africa?

(A) the expansion of political violence and civil wars across Africa

(B) the expansion of Pan-Africanism

(C) the Scramble for Africa

(D) the demand of colonial nations for independence and decolonization

Questions 24 - 26 refer to the late 20th century photograph by Stuart Franklin.

© Stuart Franklin/Magnium Photos (TheGuarian.com)

24. Conditions like those shown in the image contributed most directly to which of the following?

(A) an increase in Socialist sentiments

(B) the passage of democratic reforms under Communism and Dictatorship

(C) a decline in the use of violence against citizens

(D) an increase in media attention for civil disobedience

25. The conditions shown in the image depict which of the following trends in the late 20th century?

(A) the isolation of the urban working class

(B) China's growing military

(C) the corruption of Communist governments

(D) the growing gap between Communist government and its people

26. Advocates for individuals such as those shown in the image would have most likely agreed with which of the following perspectives?

(A) a strong economy should be well regulated to benefit all

(B) the people should be represented in the military

(C) the government should be responsive to the needs of the people

(D) Laissez-Faire style capitalism will lead to economic benefits for workers

Questions 27 - 29 refer to the excerpt below.

"That the various forms of epidemic, endemic, and other disease...chiefly amongst the labouring classes...produced by decomposing animal and vegetable substances, by damp and filth, and close and overcrowded dwellings prevail amongst the population in every part of the kingdom, whether dwelling in separate houses, in rural villages, in small towns, in the larger towns...Contaminated London drinking water containing various micro-organisms, refuse (trash), and the like. The high prosperity in respect to employment and wages, and various and abundant food, have afforded to the labouring classes no exemptions from attacks of epidemic disease...That the formation of all habits of cleanliness is obstructed by defective supplies of water.That the annual loss of life from filth and bad ventilation are greater than the loss from death or wounds in any wars in which the country has been engaged in modern times.

Sir Edwin Chadwick, Inquiry Into Sanitary Conditions of the Working Class of England, 1842

27. Which of the following aspects of Chadwick's description expresses a major change in Europeans' views of sanitation?

(A) London water has been contaminated

(B) religion has a role to play in ensuring sanitation codes

(C) every worker should be responsible for his/ her own hygiene

(D) the government should ensure basic sanitation standards for workers

28. Chadwick's ideas are most directly a reaction to the

(A) the increase in social tensions and abuse of industrial workers

(B) growing middle class of urban workers

(C) the increase in demand for raw materials for the new factories

(D) the mechanics of the factory system

29. Chadwick's position regarding the sanitary conditions of the workers was most strongly supported by which of the following?

(A) Anti-imperialists

(B) Factory owners

(C) Members of Parliament

(D) Communists

Questions 30-33 refer to the excerpt below.

"Reading this proclamation, I have concluded that the Westerners are petty indeed. It is impossible to reason with them because they do not understand larger issues as we understand them in China. There is not a single Westerner versed in Chinese works, and their remarks are often incredible and ridiculous. To judge from this proclamation, their religion is no different from other small, bigoted sects of Buddhism or Taoism. I have never seen a document which contains so much nonsense. From now on, Westerners should not be allowed to preach in China, to avoid further trouble."

Qing Emperor Kangxi, Decree on Christianity, 1721 

30. The decree imposed by Kangxi was a response to the?

(A) controversy regarding support for various minority religions

(B) debate over the legality of Confucianism within the Qing empire

(C) debate over the treatment of Christian missionaries in East Asia

(D) the dispute over monotheism

31. The ideas expressed in Kangxi's decree most strongly influenced which event during the Qing era?

(A) The Taiping Rebellion

(B) the fall of Empress Dowager Xi

(C) the Opium Wars

(D) The Boxer Rebellion

32. Which of the following groups most strongly opposed Kangxi's point of view in the decree?

(A) The Chinese peasants and farmers

(B) the Confucian scholar-gentry

(C) Christian missionaries

(D) European merchants

33. Which of the following developments demonstrated the strongest continuity with the idea expressed in the decree?

(A) The increase in social tensions between the government and peasants

(B) The development of Christianity as a minority in China

(C) The decline of the scholar gentry class

(D) The spread of Christianity throughout parts of southern China

Questions 34 - 36 refer to the excerpts below.

The Columbian Exchange, is one of the more spectacular and significant ecological events of the past millennium. When Europeans first touched the shores of the Americas, Old World crops such as wheat, barley, rice, and turnips had not traveled west across the Atlantic, and New World crops such as maize, white potatoes, sweet potatoes, and manioc had not traveled east to Europe. In the Americas, there were no horses, cattle, sheep, or goats, all animals of Old World origin. Except for the llama, alpaca, dog, a few fowl, and guinea pig, the New World had no equivalents to the domesticated animals associated with the Old World, nor did it have the pathogens associated with the Old World’s dense populations of humans...Among these germs were those that carried smallpox, measles, chickenpox, influenza, malaria, and yellow fever.

Alfred Crosby, The Columbian Exchange, 1978

34. The import of Old World crops to the New World transformed American society mostly by?

(A) diversifying diets and thereby triggering a population increase

(B) encouraging the privatization of lands

(C) improving relations between Europeans and native inhabitants

(D) encouraging warfare between European powers

35. The patterns described in the excerpt most directly foreshadowed which of the following developments?

(A) the spread of corn from South America to North America

(B) the gradual transition from a feudal to capitalist society

(C) The population decline of Native Americans

(D) the emergence of racially mixed populations in the Americas

36. The trends described by Crosby most closely corresponds with which of the following major historical developments in the Atlantic world?

(A) The rise of the Triangle Trade

(B) The Manila Galleons

(C) The growth of imperialism in Africa

(D) The Plague

Questions 37 - 39 refer to the map below.The Silk Road Trade Routes around 600 C.E.

37. The pattern of trade routes after 600 C.E. were linked most directly bywhich of the following factors?

(A) Abassid Empire's attempts to impose taxes on non-Muslims

(B) the fall of the western Roman empire

(C) the role of the Monsoon winds

(D) The rising influence of Muslim governments and merchants

38. The difference in trade route patterns from the land to sea had which of the following effects?

(A) promoted the advancement and diffusion of maritime technology

(B) a decrease in the significance of the monsoon winds

(C) an increase in conflict between the Chinese and Arab merchants

(D) an increase in trade with western Europe

39. The difference in trade patterns between the land and sea routes best explains the

(A) significance of understanding how to navigate the Monsoon winds

(B) development of religious differences between Africans, and Persians

(C) spread of the Bubonic Plague

(D) growth of religious tensions between Christians and Muslims

Questions 40 - 42 refer to the excerpt below.

"... No one ought to harm another in his life, health, liberty, or possessions... ; for without this the law could not have that which is absolutely necessary to its being a law, the consent of the society, over whom nobody can have a power to make laws but by their own consent and by authority received from them….: They must not raise taxes on the property of the people without the consent of the people.... When any one...make laws without authority, which the people are not therefore bound to obey;...and may constitute to themselves a new legislative..."

John Locke, Two Treatises on Government, 1689

40. Which of the following groups would have been most likely to support Locke's views expressed in the excerpt?

(A) Conservative landowners

(B) liberal revolutionaries

(C) Absolute monarchs

(D) members of the clergy

41. Which of the following most directly undermines Locke's claims?

(A) king's rarely give up their power

(B) the development of the Trans-Atlantic slave network

(C) many colonists engaged in forms of resistance to taxation

(D) a majority of Enlightenment philosophes were deists

42. In the 18h century, the views expressed by Locke most directly contributed to

(A) increased social tension between landlords and peasants

(B) the development of Feudalism in Russia

(C) the rise of social divisions within European society

(D) the drafting of the American Declaration of Independence

Questions 43 - 44 refer to the excerpt below.

"Now the Persian nation is made up of many tribes. Those which Cyrus assembled and persuaded to revolt from the Medes were the principal ones on which all the others are dependent. These are the Pasargadae, the Maraphians, and the Maspians, of whom the Pasargadae are the noblest. The Achaemenidae, from which spring all the Perseid kings, is one of their clans... The customs which I know the Persians to observe are the following: they have no images of the gods, no temples nor altars, and consider the use of them a sign of folly. This comes, I think, from their not believing the gods to have the same nature with men, as the Greeks imagine. Their wont, however, is to ascend the summits of the loftiest mountains, and there to offer sacrifice to Zeus, which is the name they give to the whole circuit of the firmament. They likewise offer to the sun and moon, to the earth, to fire, to water, and to the winds. These are the only gods whose worship has come down to them from ancient times."

Herodotus, On the Customs of the Persians, 430 B.C.E.

43. The excerpt would be most useful to historians as a source of information about which of the following?

(A) the similarities and difference of Greek and Persian gods

(B) the nature of ruler and citizen in ancient Persia

(C) the interaction between Greeks and Persians in the classical age

(D) the role that religion has played historically in Persian culture

44. Which of the following was a major contrast between the customs of the Persians and those of the Greeks?

(A) the Persians have no images of gods nor altars for religious worship

(B) Greeks had less gods than Persians

(C) the Greek economy was based on agriculture

(D) the Persians offered sacrifice to their gods

Questions 45 - 47 refer to the excerpt below.

" When Motecuhzoma [Montezuma] had given necklaces to each one, Cortés asked him: "Are you Motecuhzoma?

Are you the king? Is it true that you are the king Motecuhzoma?"And the king said: "Yes, I am Motecuhzoma."

Then he stood up to welcome Cortés; he came forward, bowed his head low and addressed him...

The Spaniards attacked the musicians first, slashing at their hands and faces until they had killed all of them. The

singers-and even the spectators- were also killed. This slaughter in the Sacred Patio went on for three hours. Then

the Spaniards burst into the rooms of the temple to kill the others: those who were carrying water, or bringing fodder

for the horses, or grinding meal, or sweeping, or standing watch over this work. The Sun had treacherously

murdered our people on the twentieth day after the captain left for the coast. We allowed the Captain to return to the

city in peace. But on the following day we attacked him with all our might, and that was the beginning of the war."

An Aztec Account of the Conquest of Mexico, c. 1520's

45. The Aztec account most directly illustrated the debates about which of

the following issues in America?

(A) The process of breaking colonial laws

(B) The issuing of the "Requirement"

(C) The colonization of the Caribbean

(D) The treatment of "the Other" upon first contact in the New World

46. The actions of the Spanish described in the Aztec account contributed to which of the following economic developments within the first century of colonization in the Americas ?

(A) the establishment of the encomienda system

(B) the conquest of the Maya by Cortez

(C) the rise of anti colonial governors caused by the mistreatment of natives

(D) the reliance on the Trans-Atlantic Slave Trade for labor

47. Efforts by the Aztecs to defeat the Spanish militarily ultimately failed because

(A) the more powerful tribes in the area allied with Spain against the Aztecs

(B) the Aztecs were divided among various Mexican tribes

(C) Aztecs had no immunities to disease while the Europeans had superior metallurgy and technology

(D) They hesitated to attack the Spanish when they had the chance

Questions 48 - 50 refer to the excerpt below.

“The most celebrated system of jurisprudence known to the world begins, as it ends, with a Code. From the commencement to the close of its history, the expositors of Roman Law consistently employed language which implied that the body of their system rested on the Twelve Decemviral Tables, and therefore on a basis of written law. Except in one particular, no institutions anterior to the Twelve Tables were recognised at Rome. The theoretical descent of Roman jurisprudence from a code, the theoretical ascription of English law to immemorial unwritten tradition, were the chief reasons why the development of their system differed from the development of ours. Neither theory corresponded exactly with the facts, but each produced consequences of the utmost importance.

Henry Maine, Lawyer and Historian, Ancient Law, 1861

48. The Twelve Tables were significant in the history of law because they

(A) were the first successful attempt of the Romans to codify law

(B) were the first patriarchal laws

(C) discriminated against slaves, women, and landless peasants

(D) promoted the study of law among the Plebian class of society

49. The Twelve Tables most directly reflected?

(A) conflicts from the imperial Roman period

(B) the political problems caused by invading groups from Europe

(C) social tensions between the Patrician and Plebian classes

(D) the conflict between urban and rural populations

50. Which of the following evidence would best support Maine's argument in the excerpt?

(A) Political documents in modern law codes that reflect the principles of the Twelve Tables

(B) Testimony from government officials that the Twelve Tables was influential on modern politics

(C) Narratives from the lives of Roman politicians who drafted the Twelve Tables

(D) Statistical data revealing the number of Plebians and Patricians tried as a result of the Twelve Tables

Questions 51 - 53 refer to the political cartoon below.

Courtesy of Wikimedia Commons

51. The political cartoon was intended to

(A) advocate for the Chinese to stand up to the imperialists

(B) critique how imperialism operated in China

(C) show how the Europeans were divided along political lines

(D) promote women's equality during the era of high imperialism

52. The poster most directly reflects the

(A) emergence of Japan as an imperial power

(B) the strength of the German and French militaries

(C) the violence caused by cross-cultural trade

(D) inability of China to resist the imperialists

53. Which of the following represents a later example of the actions demonstrated in the cartoon ?

(A) The Japanese invasion of Manchukuo in the 1930's

(B) The Chinese Civil war between the Nationalists and Communists from 1911 to 1945

(C) The Chinese Communist Revolution in 1945

(D) the growing tide of nationalism and decolonization in Asia

Questions 54 - 55 refer to the excerpt below.

"Good evening. Today, our fellow citizens, our way of life, our very freedom came under attack in a series of deliberate and deadly terrorist acts. The victims were in airplanes, or in their offices; secretaries, businessmen and women, military and federal workers; moms and dads, friends and neighbors. Thousands of lives were suddenly ended by evil, despicable acts of terror... America was targeted for attack because we’re the brightest beacon for freedom and opportunity in the world. And no one will keep that light from shining... The search is underway for those who are behind these evil acts. I’ve directed the full resources of our intelligence and law enforcement communities to find those responsible and to bring them to justice. We will make no distinction between the terrorists who committed these acts and those who harbor them... America and our friends and allies join with all those who want peace and security in the world, and we stand together to win the war against terrorism."

Former U.S. President George W. Bush, Address to the Nation, 9/11/2001

54. Which of the following actions by the Bush Administration best reflects the ideas about winning the war againstterrorism expressed in the excerpt ?

(A) the establishment of an American command center in Africa

(B) the decision to give U.S. foreign aid to nations that support American interests

(C) the U.S. invasions of Afghanistan and Iraq

55. The ideas expressed by Bush in the excerpt were most similar to those of which twentieth century world leader?

(A) Ho Chi Minh

(B) Franklin Delano Roosevelt

(C) Woodrow Wilson

(D) Joseph Stalin

(D) the attempted overthrow of the Venezuelan President, Hugo Chavez

END OF PART AIF YOU FINISH BEFORE TIME IS CALLED, YOU MAY CHECK YOUR WORK ON PART A. DO NOT GO ON TO PART B UNTIL YOU ARE

TOLD TO DO SO.

AP__________________________

Sample AP World History2017 Free-Response Questions__________________________

Directions: Read each passage carefully. Answer on a separate paper using complete sentences- bullet points are not acceptable. 4 questions; 50 minutes.

1) Answer a, b, and c.

a) Briefly explain ONE important similarity between Egyptian and Mesopotamian religion in the ancient period prior to 500 B.C.

b) Briefly explain ONE important difference between Egyptian and Mesopotamian religion in the ancient period prior to 500 B.C.

c)Briefly explain ONE historical factor that accounts for the difference that you indicated in b.

Use the map below to answer all parts of the question that follows.

Source: http://sherylsbuddism.weebly.com/spread-of-buddhism.html

2) Using the map, answer a, b, & c.a) Briefly explain how ONE major historical factor contributed to the information depicted on the map.

b) Briefly explain ONE specific historical effect that resulted from the information depicted on the graph.

c) Explain ONE way in which Asian society or politics was affected from the information depicted on the map.

Directions: Read each passage carefully. Answer on a separate paper using complete sentences- bullet points are not acceptable. 4 questions; 50 minutes.

Use the chart below to answer all parts of the question that follows.

Source: Independent research on European Dark Ages

1) BASED ON THE GRAPHa) Briefly explain how ONE major historical factor contributed to the change depicted on the graph.

b) Briefly explain ONE specific historical effect that resulted from the change depicted on the graph.

c) Briefly explain how ONE person, event, or development from the period 500-1500 that is not explicitly mentioned on the graph could be used to support the information shown on the graph.

Use the passages below to answer all parts of the question that follows.

" One method of delivery alone remains to us; which is simply this...facts... But not only is a greater abundance of experiments to be sought for and procured, and that too of a different kind from those hitherto tried; an entirely different method, order, and process for carrying on and advancing experience must also be introduced... For first of all we must prepare a Natural and Experimental History, sufficient and good; and this is the foundation of all; for we are not to imagine or suppose, but to discover, what nature does or may be made to do... Therefore in the third place we must use Induction, true and legitimate induction, which is the very key of interpretation. [* induction = starts with specific facts to draw more general conclusions.]

Sir Francis Bacon: First Book of Aphorisms (1620).

"Therefore . . . , invoking the most holy name of our Lord Jesus Christ and of His Most Glorious Mother Mary, We pronounce this Our final sentence: We pronounce, judge, and declare, that you, the said Galileo . . . have rendered yourself vehemently suspected by this Holy Office of heresy, that is, of having believed and held the doctrine (which is false and contrary to the Holy and Divine Scriptures) that the sun is the center of the world, and that it does not move from east to west, and that the earth does move, and is not the center of the world..."

Documents in the Case of Galileo: Indictment, of 1633.

4. a) Describe ONE important difference between the views of science and society expressed in the two passages.

b) For EACH of the passages, identify and explain ONE factor (such as a historical development, an intellectual or philosophical trend, or a religious belief) that likely informed the view of science and society expressed in the passage.

c) Identify ONE specific example of scientific discovery from 1450 to 1750 and briefly explain how the example was supported or opposed by the view of either document.

END OF SECTION 1

END OF SECTION 1IF YOU FINISH BEFORE TIME IS CALLED, YOU MAY CHECK YOUR WORK ON PART A. DO NOT GO ON TO PART B UNTIL YOU ARE

TOLD TO DO SO.

WORLD HISTORY

SECTION II

Total Time-- 1 hour, 30 minutes

Question 1 (Document-Based Question)

Suggested reading and writing time: 55 minutes

It is suggested that you spend 15 minutes reading the documents and 40 minutes writing your response.

Note: You may begin writing your response before the reading period is over.

Directions: Question 1 is based on the accompanying documents. The documents have been edited for the purpose of this exercise.

In your response you should do the following:

Thesis: Present a thesis that makes a historically defensible claim and responds to all parts of the question. The thesis must consist of one or more sentences located in one place, either in the introduction or the conclusion.

Argument Development: Develop and support a cohesive argument that recognizes and accounts for historical complexity illustrating relationships among historical evidence such as contradiction, collaboration, and/or qualification.

Use of the Documents: Explain the significance of the author's point of view, author's purpose, historical context, and/or audience for at least four documents.

Contextualization: Situate the argument by explaining the broader historical events, developments, or processes immediately relevant to the question.

Outside Evidence: Provide an example or additional piece of specific evidence beyond those found in the documents to support or qualify the document.

Synthesis: Extend the argument by explaining the connections between the argument and ONE of the following.

A development in a different historical period, situation, era, or geographical area. A course theme and/ or approach to history that is not the focus of the essay (such as political,

economic, social, or intellectual history) A different discipline or field of inquiry (such as economics, government and politics, art history,

or anthropology).

1. Explain the causes of the rise of the Indian National Congress during the period 1857-1947.

Document 1

Source: A photo of Indian National Congress leaders, Jawaharlal Nehru and Mohandas Gandhi

Document 2

MARCH 11, 1930 

MY DEAR JAWAHARLAL,

  It is nearing 10 p.m. now. The air is thick with the rumour that I shall be arrested during the night. I have not wired to you especially because the correspondents submit their messages for approval and everybody is working at top speed. There was nothing special to wire about.

  Things are developing extraordinarily well. Offers of volunteers are pouring in. The column will proceed with the march even though I may be arrested. If I am not, you may expect wires from me, otherwise I am leaving instructions.

  I do not know that I have anything in particular to say. I have written enough. I gave a final message this evening to a vast crowd that gathered for prayer on the sands.

  May God keep you and give you strength to bear the burden.

With love to you all,Bapu (Gandhi)

- Letter from Gandhi to Nehru

Document 3

...We have a stronger weapon, a political weapon, in boycott. We have perceived one fact, that

the whole of this administration, which is carried on by a handful of Englishmen, is carried on

with our assistance. We are all in subordinate service. This whole government is carried on with

our assistance and they try to keep us in ignorance of our power of cooperation between

ourselves by which that which is in our own hands at present can be claimed by us and

administered by us.

The point is to have the entire control in our hands. I want to have the key of my house,

and not merely one stranger turned out of it. Self-government is our goal; we want a control over

our administrative machinery. We don't want to become clerks and remain [clerks]. At present,

we are clerks and willing instruments of our own oppression in the hands of' an alien

government, and that government is ruling over us not by its innate strength but by keeping us

in ignorance and blindness to the perception of this fact. Professor Seeley shares this view.

Every Englishman knows that they are a mere handful in this country and it is the business of

every one of them to befool you in believing that you are weak and they are strong. This is

politics. We have been deceived by such policy so long.

What the new party wants you to do is to realize the fact that your future rests entirely in

your own hands. If you mean to be free, you can be free; if you do not mean to be free, you will

fall and be for ever fallen. So many of you need not like arms; but if you have not the power of

active resistance, have you not the power of self-denial and self-abstinence in such a way as

not to assist this foreign government to rule over you? This is boycott and this is what is meant

when we say, boycott is a political weapon. We shall not give them assistance to collect

revenue and keep peace. We shall not assist them in fighting beyond the frontiers or outside

India with Indian blood and money. We shall not assist them in carrying on the administration of

justice. We shall have our own courts, and when time comes we shall not pay taxes. Can you

do that by your united efforts? If you can, you are free from tomorrow.

- Indian politician's address to Indian National Congress (1907)

Document 4

DEHRA DUN JAIL, JULY 11, 1932

TO: THE SUPERINTENDENT, DISTRICT JAIL, DEHRA DUN

DEAR SIR,

  You were good enough to show me today the reply of the Officiating Inspector General of Prisons to my letter dated the 22nd June. I am informed therein that, in the course of an interview with Mr. R.S. Pandit in the Allahabad District Jail on May 27th, my wife handed a letter to Mr. Pandit, and the Jailer not allowing this, my mother "used insulting language to the Jailer and was impertinent."

...Apart from the insult to Mr. Pandit, the Jailer's behavior was an affront to my mother and wife. My mother hardly spoke to him.

  Three days later, on May 30th, I had my usual fortnightly interview with my mother, wife and daughter in the Bareilly District Jail. I was then informed of what had happened. I was surprised to learn that anyone should have behaved so discourteously to my mother and I expected some expression of regret from the Jail officials for what had occurred. Instead of that, I now find that the Government have chosen to punish my mother and wife. I presume this has been done on some statement made to them by the Jailer. No reference was made, so far as I am aware, to my mother or my wife to find out what had happened. Without any further enquiry or effort to find out the truth, the Government have not hesitated to insult my mother and wife, and have done so in such a way as to cause the maximum inconvenience to all parties concerned.

  It may be that it is an offence under the jail regulations to show a school report about one's children. If Government wish to treat even this as worthy of punishment, I have no grievance. Nor shall I object if my interviews are stopped for a month or a year. I have not come to prison for the sake of my health or for pleasure.

  But there are certain matters which I cannot pass in silence. I cannot tolerate even the suspicion of an affront or insult to my mother. I have noticed with deep regret that Government have not shown my mother the courtesy which I would have expected from them under any circumstances. For the Inspector-

General to say that my mother "used insulting language to the Jailer and was impertinent" shows that he is strangely lacking in a sense of proportion and knows little of Indian society.

  On no account am I prepared to take the slightest risk of further insult to my mother and wife. Under the circumstances, the only course open to me is not to have any interviews, so long as I do not feel that such interviews can be had with dignity and with no fear of discourtesy to those who come to see me. I am therefore informing my people not to take the trouble to come for interviews with me in future, even after the month of punishment is over...

  

Yours faithfully,

Jawaharlal Nehru

Document 5

Source: A newspaper article about the Amritsar Massacre published on April 19, 1919.

Document 6

We have now before us the data for understanding, at least in a measure, the meaning of the "New National Movement in India." It is the awakening and the protest of a subject people. It is the effort of a nation, once illustrious, and still conscious of its inherent superiority, to rise from the dust, to stand once more on its feet, to shake off fetters which have become unendurable. It is the effort of the Indian people to get for themselves again a country which shall be in some true sense their own, instead of remaining, as for a century and a half it has been, a mere preserve of a foreign power,—in John Stuart Mill's words, England's "cattle farm." The people of India want the freedom which is their right,—freedom to shape their own institutions, their own industries, their own national life. This does not necessarily mean separation from Great Britain; but it does mean, if retaining a connection with the British Empire, becoming citizens,and not remaining forever helpless subjects in the hands of irresponsible masters. It does mean a demand that India shall be given a place in the Empire essentially like that of Canada or Australia,with such autonomy and home rule as are enjoyed by these free, self-governing colonies. Is not this demand just? Not only the people of India, but many of the best Englishmen, answer unequivocally, Yes! In the arduous struggle upon which India has entered to attain this end (arduous indeed her struggle must be, for holders of autocratic and irresponsible power seldom in this world surrender their power without being compelled) surely she should have the sympathy of the enlightened and liberty-loving men and women of all nations.

-Atlantic Monthly magazine article (1909)

Document 7

We are meeting today in our session after fifteen months. The last session of the All-India Muslim League took place at Patna in December 1938. Since then many developments have taken place. ...But a great deal yet remains to be done. I am sure from what I can see and hear that the Muslim India is now conscious, is now awake, and the Muslim League has by now grown into such a strong institution that it cannot be destroyed by anybody, whoever he may happen to be. Men may come and men may go, but the League will live for ever.

[[10] He [Gandhi] is fighting the British. But may I point out to Mr. Gandhi and the Congress that you are fighting for a Constituent Assembly which the Muslims say they cannot accept; which, the Muslims say, means three to one; about which the Mussalmans say that they will never be able, in that way by the counting of head, to come to any agreementwnt which will be real agreement from the hearts, which will enable us to work as friends; and therefore this idea of a Constituent Assembly is objectionable, apart from other objections. But he is fighting for the Constituent Assembly, not fighting the Mussalmans at all! ...

[[11]] So he wants the Constituent Assembly for the purpose of ascertaining the views of the Mussalmans; and if they do not agree then he will give up all hopes, but even then he will agree with us. (Laughter.) Well, I ask you. ladies and gentlemen, is this the way to show any real genuine desire, if there existed any, to come to a settlement with the Mussalmans? (Voices of no, no.) Why does not Mr. Gandhi agree, and.I have suggested to him more than once and I repeat it again from this platform, why does not Mr. Gandhi honestly now acknowledge that the Congress is a Hindu Congress, that he does not represent anybody except the solid body of Hindu people? Why should not Mr. Gandhi be proud to say. "I am a Hindu. Congress has solid Hindu backing"? I am not ashamed of saying that I am a Mussalman. (Hear, hear and applause.) I am right and I hope and I think even a blind man must have been convinced by now that the Muslim League has the solid backing of the Mussalmans of India (Hear, hear.) Why then all this camouflage? Why all these machinations? Why all these methods to coerce the British to overthrow the Mussalmans? Why this declaration of non-cooperation? Why this threat of civil disobedience? And why fight for a Constituent Assembly for the sake of ascertaining whether the Mussalmans agree or they do not agree? (Hear, hear.) Why not come as a Hindu leader proudly representing your people, and let me meet you proudly representing the Mussalmans? (Hear, hear and applause.)...

As regards other matters, we are still negotiating and the most important points are: (1) that no declaration should be made by His Majesty's Government with regard to the future constitution of India without our approval and consent (Hear, hear, and applause) and that no settlement of any question should be made with any party behind our back (Hear, hear) unless our approval and consent is given to it.

-Muhammad Ali Jinnah, Indian Muslim leader: Address to the Muslim League (1940)

END OF DOCUMENTS FOR QUESTION 1

WORLD HISTORY

SECTION II

Total Time- 1 hour, 30 minutes

Question 2 or Question 3

Suggested writing time: 35 minutes

Directions: Choose EITHER question 2 or question 3.

In your response you should do the following.

Thesis: Present a thesis that makes a historically defensible claim and responds to all parts of the question. The thesis must consist of one or more sentences located in one place, either in the introduction or conclusion.

Application of Historical Thinking Skills: Develop and support an argument that applies historical thinking skills as directed by the question.

Supporting the Arguments with Evidence: Utilizes specific examples of evidence to fully and effectively substantiate the stated thesis or relevant argument.

Synthesis: Extend the argument by explaining the connections between the argument and ONE of the following.

A development in a different historical period, situation, era, or geographical area. A course theme and/or approach to history that is not the focus of the essay (such as political,

economic, social, cultural, or intellectual history). A different discipline or field of inquiry (such as economics, government and politics, art history,

or anthropology).

3) Evaluate the extent to which the Enlightenment (1600's-1700's) marked a turning point in the triumph of democratic revolutions in world history, analyzing what changed and what stayed the same from the period before the Enlightenment to the period after it. (Historical Thinking Skill: Periodization)

or

2.) Evaluate the extent to which the Industrial Revolution marked a turning point in the history of Europe's rise to material and commercial wealth.

In the development of your argument, explain what changed and what stayed the same from the period immediately before the industrial revolution to the period immediately following it.. (Historical Thinking Skill: Periodization)

STOP---END OF EXAM

AP__________________________

Sample AP World History

2017 Multiple-Choice Answers

__________________________

Answers to Multiple-Choice Questions

1. (C) Trans-Atlantic Exchanges

Equiano describes being sold as a slave across the Atlantic Ocean.

2. (C) The abolition of international slavery

Equiano's writings influenced abolitionist movements in Europe.

3. (B) The Triangle Trade

The Triangle Trade is another term for the Trans-Atlantic slave trade.

4. (D) The urbanization of Europe during the Industrial Revolution

In Europe, industrialization, urbanization, and population growth were linked.

5. (A) Asia

The most populous nations are India, and China- two Asian countries.

6. (A) An increase in tensions between social classes

Tensions between capitalists and factory workers led to the rise of Communism.

7. (B) The beginning of an industrialized economy

The world's population has increased exponentially since Europe industrialized.

8. (C) women shouldn't enjoy full and equal rights with men

She is arguing for equal rights for women, contrary to popular opinion.

9. (A) The 19th Amendment to the U.S. Constitution, granting women the right to vote

The women's suffrage movement was influenced by the works of Olympe de Gouges.

10. (B) Radical supporters of the French Revolution

Many women resented the lack of change in status even after the revolution.

11. (D) The Mongols ability to conquer previously wealthy and powerful empires

The writings were in response to the rise of the Mongols after Gengis Khan.

12. (A) The military and political traditions of central Asian nomads

The political traditions of central Asian nomads allowed for brutality and slaughter.

13. (D) The spread of disease and eventual collapse of the Silk Road

The Mongols facilitated the spread of disease, leading to the deaths of millions.

14. (C) the rise of Zionism promoting Israel as a homeland for the Jews

The Balfour Declaration boosted calls of Zionism, and immigration to Israel.

15. (C) national boundaries

Zionism was an example of post war nationalism. The British supported Zionism.

16. (A) U.N. Israel/Palestine Partition Plan of 1947

The partition in 1947 was a continuity because it also divided Israel by Arabs and Jews.

17. (A) Sykes-Picot Agreement, granting Britain control of Mandate Palestine

Sykes-Picot gave control to Britain whereas the Balfour document favored Jews in Palestine.

18. (B) highlighting the decline of the natives as part of colonization

Most Spanish weren't concerned with the treatment of the native Americans.

19. (C) encourage the improved treatment of the natives

He's writing about the horrible loss of life in order to improve the situation.

20. (A) the treatment of slaves as part of the Trans-Atlantic Slave trade

The brutal demise of the Indians was similar to that of the slaves who came to the Americas.

21. (B) breaking free from the chains of colonization to establish an independent nation

Both passages deal with both authors' struggles for independence in a post-colonial world.

22. (A) show the state along with the people will influence political affairs

Sun Yet Sen was saying that the state would play an important role in society.

23. (D) the demand of colonial nations for independence and decolonization

Kenyatta's demands best reflect his early wants and desires for independence.

24. (B) the passage of democratic reforms under Communism and Dictatorship

The man was resisting communism in China, but influenced Soviet as well.

25. (D) the growing gap between Communist government and its people

In the late 1980's, the Soviet Union was crumbling and becoming more democratic.

26. (C) the government should be responsive to the needs of the people

The picture represented the people protesting against their governments.

27. (D) the government should ensure basic sanitation standards for workers

Before, nobody cared about hygiene and sanitation. Now, it's a public matter.

28. (A) the increase in social tensions and abuse of industrial workers

His writings showed sympathy for workers, and led to some of the first labor laws ever.

29. (D) Communists

Since he was concerned with the treatment of workers, communists should support him.

30. (C) debate over the treatment of Christian missionaries in East Asia

At first, Christianity was allowed in China. Then, Kangxi prohibited it for various reasons.

31. (D) The Boxer Rebellion

The Kangxi decree influenced the Boxer Rebellion in that both were anti-Christian.

32. (C) Christian missionaries

The Christian missionaries would not be able to support Kangxi's decree without a fight.

33. (B) The development of Christianity as a minority in China

The fact that China didn't officially promote Christianity led to its minority status.

34. (A) diversifying diets and thereby triggering a population increase

With the transfer of wheat and barley, the diets in the Americas changed.

35. (C) The population decline of Native Americans

The end of the passage mentions diseases, foreshadowing the decline of the Indians.

36. (A) The rise of the Triangle Trade

The Triangle Trade or Trans-Atlantic slave trade was part of the Columbian Exchange.

37. (D) The rising influence of Muslim governments and merchants.

The Abassids and other merchants sold goods from Aleppo to Malacca.

38. (A) promoted the advancement and diffusion of maritime technology

The spread of maritime technology like the compass was facilitated by sea routes.

39. (A) significance of understanding how to navigate the Monsoon winds

Explains why Indians needed to know the monsoon winds more than Mongolians.

40. (B) liberal revolutionaries

Liberal revolutionaries looking to establish democratic governments read Locke.

41. (B) the development of the Trans-Atlantic slave network

Locke said that no one ought to be deprived of life, yet America was founded on slavery.

42. (D) the drafting of the American Declaration of Independence

Locke influenced Thomas Jefferson when he wrote the Declaration of Independence.

43. (D) the role that religion has played historically in Persian culture

The passage is useful to read about how Greeks viewed Persian religion.

44. (A) the Persians have no images of gods nor altars for religious worship

The Greeks had many altars and places of worship for the gods like the Parthenon.

45. (D) The treatment of the Other upon first contact in the New World

The issue revolves around the treatment of natives by colonizers, and vice versa.

46. (A) the establishment of the encomienda system

In the earliest days, Indian labor was used on the encomienda system- slaves came later.

47. (C) Aztecs had no immunities to disease while the Europeans had superior metallurgy and

technology

As Jared Diamond explains, Europeans had the advantages of guns, germs, and steel.

48. (A) were the first successful attempt of the Romans to codify law

The Romans had many social customs but before, but it hadn't yet been codified.

49. (C) social tensions between the Patrician and Plebian classes

The lower class, Plebains, wanted to end the corruption and have a say in government.

50. (A) Political documents in modern law codes that reflect the principles of the Twelve

Tables

The best evidence to compare to modern day law would be to have cited examples.

51. (B) critique how imperialism operated in China

The document is using artistic devices to criticize how Europeans just took over China.

52. (D) inability of China to resist the imperialists

The poster shows the Chinese man standing behind the table with his arms up- defeated.

53. (A) The Japanese invasion of Manchukuo in the 1930's

The Japanese imperialist is at the table, just like the attack on China that led to WWII.

54. (C) the U.S. invasions of Afghanistan and Iraq

The invasions of Afghanistan and Iraq are policy decisions reflected in the speech.

55. (B) Franklin Delano Roosevelt

Much like Bush, F.D.R. declared a war after being attacked at Pearl Harbor.

AP__________________________

Sample AP World History2017 Free-Response Answers

__________________________

SECTION I, PART B: SHORT-ANSWER QUESTIONS (EXPLAINED)

Question 1

This question deals with comparing and contrasting religions in ancient Mesopotamia and Egypt.

a) One similarity is that they both built architectural monuments that were reserved for religious purposes. In Sumer, the inhabitants built ziggurat as a form of temple worship. In Egypt, the pyramids served religious purposes and burial grounds for pharaohs.

b) The main difference between the groups was their beliefs in the number of gods. In Mesopotamia, the Hebrews were the first group to practice monotheism- the belief in one G*d. On the other hand, the Egyptians were polytheistic, believing in many gods. The Egyptians had a belief in the afterlife, and worshipped many gods like Osiris and Isis whereas the Jewish people worshipped only G*d.

c) One factor that accounts for the main difference is that in the ancient era King David united Israel. Jerusalem became the capital and most holy city for all Jews. This led to the full establishment of Jewish kingdoms in Israel before Nebuchadnezar destroyed the Temple.

SECTION I, PART B: SHORT-ANSWER QUESTIONS (EXPLAINED)

Question 2

This question revolves around the map of the spread of Buddhism during the Classical era.

a) There are many reasons for the spread of Buddhism during the time period. Buddhism began in India before spreading throughout Asia. In Afghanistan, the Kushan Empire promoted Buddhism and facilitated trade along the Silk Road. Until the 21st century, there were giant Buddhist statues in Bamyan, Afghanistan that were left over from the Kushan era. In China, Faxian and Xuanzang in particular helped spread Buddhism into China from India. They both went to India to learn about Buddhism before spreading their teachings back in China. In Thailand, the followers practiced a more conservative form of Buddhism, known as Theravada. In Korea and Japan, they were both influenced by missionaries. The Silla Kingdom in Korea promoted Buddhism as its official religion. More generally, the Silk Road played a huge role in the spread of Buddhism. Most of the arrows on the map are consistent with the trade routes of the Silk Road.

b) The biggest social effect has been that Southeast Asia is home to the world's largest populations of Buddhist societies. They also have the highest percentages of Buddhists in their countries. For example, China, Korea, Burma, Thailand, and Japan all have large numbers of Buddhists in their countries. They are in many ways Buddhist societies. In India, Ashoka built stupas and promoted Buddhist policies.

c) In India, Buddhism didn't really penetrate into the southern regions which remained mostly Hindus. Chinese society became Buddhist after Xuanzang traveled to India and brought back Buddhism into China. Even though he broke the Chinese law of leaving the country, he was welcomed back by the emperor and made a court official. The Kushan Empire in Afghanistan promoted Buddhism along the Silk Road.

SECTION I, PART B: SHORT-ANSWER QUESTIONS (EXPLAINED)

Question 3

The graph is at the center of this question. You need to analyze the increase in the output of European manuscripts (books) from 500-1500 C.E..

a) The biggest historical factor for the increase in the output of manuscripts in Europe from 500-1500 C.E. was the invention of the printing press. The printing press was created by Johann Gutenberg in 1450. Gutenberg turned the process of copying a book into a manufacturing process. Before his invention, all of the books had to be copied by hand. That explains one reason for the low output of manuscripts before the 15th century as seen on the graph. Since the books had to be copied by hand, the process was painstakingly slow.

There could have been other reasons as well like the Crusades or the Silk Road. During the Crusades, from the 11th to 14th century there is an increase on manuscripts shown on the graph. Perhaps, this was caused by the exchange of Muslims and Christians during their Holy War which was also promoted by the merchants during the 4th crusade. The Silk Road also increased the spread of literature and travel accounts. For example, Marco Polo's experiences on China were recorded and widely published in Europe.

b) The biggest historical effect was that knowledge flourished across Europe, which later spread to the rest of the world. The invention of the printing press made books much cheaper and more widespread. The levels of education and literacy in Europe have skyrocketed as a result of the printing press. Now, knowledge was for the common person and not just for the political and religious elite. The growth of literature spread across Europe during the Renaissance and has continued until the present day. Shakespeare was the most famous Renaissance author, but there were several others like Chaucer, Dante, and de Cervantes. Much of their works have become classic novels and required reading for many students today.

c) Johann Gutenberg is not mentioned on the graph; however, he is the most significant person in the increase in European manuscripts in the 15th century. He invented the printing press, which led to the huge increase in manuscripts from 250,000-500,000 in one century.

SECTION I, PART B: SHORT-ANSWER QUESTIONS (EXPLAINED)

Question 4

Here, you need to compare two passages about the role of science, religion, and society.

a) The biggest difference is that the first passage expresses a desire and need for experimental science whereas the second passage shows a belief in religion as the highest order of law. Sir Francis Bacon claims we need to have science that is based on experiments and inductive reasoning. The second passage, from Galileo's trial, shows how authorities weren't concerned with the progress of science so much as enforcing traditional Catholic beliefs about the universe. The first passage is from a scientific perspective, but the second one is from a Catholic point of view.

b) Sir Francis Bacon was most likely influenced by the Scientific Revolution. He advocated for the advancement of science over religion in order for society to progress. During the Scientific Revolution, scholars began using science to prove natural phenomenon.

The documents from Galileo's trial are clearly from a Catholic perspective. The Catholic Reformation may have played a huge role in convicting Galileo of heresy. The believed that G*d's word was supreme, and trumped scientific discovery, which was often viewed as heretical.

c) Copernicus' discovery of the Heliocentric Theory is one example of a scientific discovery that was supported by Bacon, but opposed by the Catholic authorities at Galileo's trial. Copernicus used his mathematical models to determine that the sun was at the center of the universe. The Catholic church explicitly opposed that belief. Galileo was on trial specifically because he "held the doctrine that the sun is the center of the world." That is exactly what Galileo was on trial for, holding form to Copernicus' belief. The work of Copernicus was consistent with Bacon's belief that society should be based on inductive reasoning.

SECTION II, PART A: DOCUMENT BASED QUESTION (EXPLAINED)

Document 1

This is a photo of Mohandas Gandhi and Jawaharlal Nehru sitting together. Both of them were independence leaders of India involved in the Indian National Congress, the main agency fighting for Indian independence. The photo shows how close they were, and how they were constantly aware of one another's positions. They look as though they are discussing and contemplating the fight for independence.

Document 2

In this letter, Gandhi writes to Nehru about how he feels he is going to be arrested that evening. Gandhi seems very calm. Most likely, he was being arrested for his boycott of British salt. The letter is dated one day before the Salt March when Gandhi and his followers collected salt despite the British monopoly. It shows how hard they had to struggle to achieve independence.

Document 3

In this Indian politician's address to the Congress, he argues the need for independence from the British. He claims that their biggest weapon is the power of boycott, a non-violent tactic. The boycott of Indian goods was a strategy of the Indian National Congress because it was non-violent yet effective in hurting the British economically. He is trying to incite the audience to demand their freedom immediately. He doesn't want a long term approach to freedom.

Document 4

This is a letter from Nehru to his jailer. It shows that Nehru was also jailed. It is similar to the Gandhi letter where he was imprisoned, but Nehru is writing to express his discontent with the mistreatment of his mother. He claims that the government insulted his mother and his wife. The document doesn't give any details of boycott activities, but it does provide is with insight into the animosity between the British and Indians during the era of Indian nationalism.

Document 5

The picture is a newspaper article from the Amritsar Massacre in Indian in 1919. It shows that the British reportedly killed 100 Indians. The Amritsar Massacre showed how the British colonization of India was a violent process. The massacre happened during the struggle for independence. It was one thing that led to greater support for Indian independence.

Document 6

In this magazine article, the author explains what the struggle for Indian independence is all about. The author argues the Indians have a right to freedom. His article is trying to inform and persuade Western audiences to support the cause for Indian independence. He says that India should be granted the same status as Canada, or Australia.

Document 7

In this document, Muhammad Ali Jinnah criticizes Gandhi and explains the need for a Muslim country. The author is addressing his own organization, the Muslim League, and telling them that Gandhi is only fighting for Hindu independence. He claims the Muslims will not be adequately represented in the new Hindu majority government if Gandhi has his way. That's why he believes they need a separate country, Pakistan, where all the Muslims can live together in peace. In the end, he remarks that the Muslims won't accept any negotiation regarding India's independence unless they approved it. His address stirred up anti-Hindu sentiments and promote Muslim nationalism in Pakistan.

SECTION II, PART B: LONG ESSAY QUESTION (EXPLAINED)

Question 2

This question asks you to write an essay evaluating the extent to which the Enlightenment marked a turning point in the history of democratic revolutions. The first thing to know is that there were three democratic revolutions in America, France, and Haiti. Now, you need to know some facts and concepts about each one that you write about.

(IDEAS & FACTS TO MENTION IN YOUR ESSAY)

Before the Enlightenment, monarchy was the rule of the day.

Europe was ruled by mostly monarchies, especially Britain and France.

France controlled the island of Haiti as a colony.

France's social system consisted of three estates or classes.

The third estate was highly taxed, but not the other estates.

All revolutions were influenced by Locke's ideas of the social contract.

Locke also argued for 3 inalienable rights- life, liberty and property.

In Haiti, the revolutionaries had a successful slave rebellion.

In America, the colonists overthrew the British monarchy.

The Americans established the first democratic republic of the era.

America established a government based on Enlightenment ideas,

Montesquieu, Voltaire and Rousseau also influenced democracy.

The Enlightenment was the intellectual roots of democratic revolutions

SECTION II, PART B: LONG ESSAY QUESTION (EXPLAINED)

Question 3

It is important to keep in mind that Europe had transitioned out of the Dark Ages during the Renaissance in the 1400's. The industrial revolution usually is referred to as roughly the time period from 1750-1900.

(IDEAS & FACTS TO MENTION IN YOUR ESSAY)

Before the Renaissance, Europe was in the Dark Ages.

The Renaissance was a rebirth of Greco-Roman culture in Europe.

Europe was becoming rich again, and catching up to the East.

After the IR, Europe became richer and more powerful than the East.

The steam engine by James Watt was a significant factor in the IR.

Europe became more urbanized, and less rural.

The living standard of the average European improved greatly.

The industrial revolution led to the era of imperialism.

Industrialization and imperialism went hand in hand.

After the era of industrialization, Europe was able to conquer China.

The Opium Wars showed Britain's industrial strength.

The IR led to the rise of a middle class in Europe as opposed to serfs.

The rise of wealthy capitalists was a result of the IR as well.

AP

__________________________ Sample AP World History

2017 Multiple Choice Questions

__________________________

WORLD HISTORY

SECTION I, Part A

Time—55 minutes

55 Questions

Directions: Each of the questions or incomplete statements below is followed by four suggested answers or completions. Select the one that is best in each case and then fill in the appropriate letter in the corresponding space

on the answer sheet.

Questions 1 - 3 refer to the excerpt below.

“Another of their (people of Mali's) good qualities is their habit of wearing clean white garments on Fridays. Even if a man has nothing but an old worn shirt, he washes it and cleans it, and wears it to the Friday service. Yet another is their zeal for learning the Koran by heart. They put their children in chains if they show any backwardness in memorizing it, and they are not set free until they have it by heart. I visited the qadi in his house on the day of the festival. His children were chained up, so I said to him, "Will you not let them loose?" He replied, "I shall not do so until they learn the Koran by heart."

Ibn Battuta, Travels in Africa, c. 1325

1) Ibn Battuta's account suggests that cultural encounters between Africans and Arabs in the 1300's was most directly shaped by

(A) interest in religious debate

(B) Trans-Atlantic Exchanges

(C) cross-cultural trade

(D) traditions and practices of Islam

2) Ibn Battuta's account contributed most directly to which of the following trends?

(A) The rise in social tensions between African rulers and their people

(B) greater political autonomy for Africa

(C) the rise and spread of Islam

(D) the rise of child soldiers in Africa

3) The experience described in the excerpt is an example of which of the following historical developments?

(A) The development of the Silk Road

(B) The urbanization of African cities

(C) The development of African literature

(D) The tension within Muslim sects

Questions 4 - 7 refer to the graph below.

GROWTH OF THE URBAN POPULATION IN CHINA, 2001 C.E..- 2009 C.E

4. Which of the following was a significant cause of the trend from 2001to 2009 shown in the graph?

(A) Active encouragement of export and manufacturing for the world economy

(B) the rising rates of poverty in the urban areas

(C) Incentives offered by European companies looking to hire foreignmigrants from colonial territories

(D) The large scale migration of Chinese into America and Europe

5. The people represented by the graph from 2001-20095 most typically worked in what type of employment?

(A) high finance and real estate

(B) government controlled industries

(C) manufacture and export

(D) support services for the tourism

6. Which of the following was a direct effect of the trend in population growth between 2001-2009 shown on the graph?

(A) urban poverty rates have skyrocketed

(B) the environment in urban areas has become polluted and hazardous

(C) the promotion of agricultural reform to increase land ownership for many

(D) The removal of government from all aspects of the economy

7. The main trend shown in the graph was most directly associated with which of the following processes occurring in the global economy at the time?

(A) The convergence of world markets in the age of globalization

(B) The abolition of tariffs for nations that support capitalist policies

(C) The increase in the number of transnational corporations from Asia

(D) The increase in exports of manufactured goods from America

Questions 8 - 10 refer to the excerpt below

"To account for and excuse the tyranny of man, many ingenious arguments have been brought forward to prove that the two sexes, in the acquirement of virtue, ought to aim at attaining a very different character: or to speak explicitly, women are not allowed to have sufficient strength of mind to acquire what really deserves the name of virtue... Women are told from their infancy, and taught by the example of their mothers, that a little knowledge of

human weakness, justly termed cunning, softness of temper, outward obedience, and a scrupulous attention to a puerile kind of propriety, will obtain for them the protection of man; and should they be beautiful, every thing else is needless, for at least twenty years of their lives..."

Mary Wollstonecraft, Vindication of the Rights of Woman, 1792

8. The sentiments conveyed in the excerpt most directly opposed the prevailing ideal in the early eighteenth century that

(A) since ancient times women have been oppressed

(B) women should be more like their mothers

(C) patriarchy was viewed as legitimate

(D) women should educate their children about the rights and responsibilities of citizenship

9. Which of the following developments in the first half of the twentieth century best represented the continuation of the ideas expressed in the excerpt?

(A) The 19th Amendment to the U.S. Constitution, granting women the right to vote

(B) Women’s support for WWI

(C) Support for outlawing the production and sale of alcohol

(D) A movement focused on religious revivals and personal conversion

10. Many female supporters of the Enlightenment opposed many philosophes over what issue?

(A) the abolition of slavery

(B) the treatment of Native Americans

(C) the failure by many liberals to grant women full equality and citizenship

(D) the decline of women's traditional roles in society

Questions 11 - 13 refer to the excerpts below.

"... The torture of a criminal during the course of his trial is a cruelty consecrated by custom in most nations...No

man can be judged a criminal until he be found guilty; nor can society take from him the public protection until it

have been proved that he has violated the conditions on which it was granted. What right, then, but that of power,

can authorize the punishment of a citizen so long as there remains any doubt of his guilt?...Either he is guilty, or not

guilty. If guilty, he should only suffer the punishment ordained by the laws, and torture becomes useless, as his

confession is unnecessary. If he be not guilty, you torture the innocent; for, in the eye of the law, every man is

innocent whose crime has not been proved..."

Cesare Beccarria, On Crimes and Punishment, 1764

“Excessive bail shall not be required, nor excessive fines imposed, nor cruel and unusual punishments inflicted.8th Amendment, U.S. Constitution, 1791

11. The excerpts were written in response to the

(A) the American Revolution

(B) the debates between conservatives and radicals over lengthy prison sentences

(C) the changing attitudes among European monarchies to reform prisons

(D) political and social acceptance of torture as a means of discipline andpunishment for suspected criminals

12. The ideas about torture and punishment expressed by both passages are most consistent with which of the following?

(A) The concept of Constitutionalism

(B) The belief in Absolutism

(C) The principle of popular sovereignty

(D) The ideas of the Enlightenment

13. The ideals expressed in both documents has accounted for which of the following features of the democratic criminal justice systems since the Enlightenment era?

(A) The increase in the rights of the accused like innocent until proven guilty

(B) the decrease in beheadings during the French Revolution

(C) the conflict of the checks and balances in a democracy

(D) The growth in conflict between the wealthy colonists and the slaves

Questions 14 - 17 refer to the excerpts below.

“1.) ...France and Great Britain are prepared to recognize and protect an independent Arab State or a Confederation of Arab States in the areas (A) and (B) marked on the annexed map, under the suzerainty of an Arab chief. That in area (A) France, and in area (B) Great Britain, shall have priority of right of enterprise and local loans. That in area (A) France, and in area (B) Great

Britain, shall alone supply advisers or foreign functionaries at the request of the Arab State or Confederation of Arab States...

Paul Cambon's letter to Edward Grey, Sykes-Picot Agreement, 1916

14. Which of the following was the most immediate result of the declaration?

(A) Arab nationalist became increasingly united over in their political strategies

(B) the rise in concern about the persecution of Jews in Europe

(C) the Arab territories of Transjordan, Palestine, Syria, and Lebanon were divided among Europeans

(D) The British refused to support the idea and granted full independence to all Arab nations

15. The decision excerpted most directly reflected a growing belief after the First World War that the Arab world's political borders should be divided according to

(A) racial boundaries

(B) religious preference

(C) national boundaries

(D) a mandate system under European influence

16. Which of the following events demonstrated the strongest similaritywith the idea expressed in the excerpt?

(A) 14 Points Plan

(B) Berlin Conference

(C) Yalta Conference

(D) Treaty of Versailles

17. The Sykes-Picot Agreement stood in contrast mostly to which of the following other decisions?

(A) Balfour Declaration

(B) 14 Points Plan

(C) Berlin Conference

(D) The establishment of the League of Nations

Questions 18 - 20 refer to the excerpt below.

" ...The trade of slaves is in a more peculiar manner the business of kings, rich men, and prime merchants, exclusive of the inferior sort of Blacks....This barbarous usage of those unfortunate wretches, makes it appear, that the fate of such as are bought and transported from the coast to America, or other parts of the world, by Europeans, is less deplorable, than that of those who end their days in their native country; for aboard ships all possible care is taken to preserve and subsist them for the interest of the owners, and when sold in America, the same motive ought to prevail

with their masters to use them well, that they may live the longer, and do them more service. Not to mention the inestimable advantage they may reap, of becoming Christians, and saving their souls, if they make a true use of their condition....

John Barbot, How Slaves Were Acquired , 1732

18. The ideas expressed in the excerpt differed from the predominant abolitionist views of the slave trade because the passage

(A) lauds the possible benefits of the slave trade

(B) highlights the decline in health for the slaves

(C) promotes new laws that would make the slave trade illegal

(D) encourages the slave owners to be good Christians

19. The excerpt best reflects an effort by Barbot to

(A) convert the slaves to Christianity

(B) persuade other abolitionists to join him

(C) encourage the improved treatment of the slaves

(D) prevent the abuses of slaveryfrom spreading to the Americas

20. Which of the following best represents a change in the years after 1800 with the account that Barbotdescribed in the excerpt?

(A) the Berlin Conference

(B) the success of the Haitian Revolution

(C) The European colonization of Africa during the era of high imperialism

(D) Simon Bolivar's struggle to unify South America into a gran Colombia

Questions 21 - 23 refer to the excerpts below.

"Damascus (city in Syria) is the paradise of the east and the rising place of its radiant beauty. The city was highly honored when Allah gave refuge to the Messiah and his Mother (Muslims believe Jesus and Mary sought refuge in Damascus) on an 'elevated spot, secure and abundant in water' {Sura 23:52). In fact wherever you look in your gaze, by Allah, they told the truth who said, "If Paradise be on earth, Damascus must be it; if it is m Heaven, Damascus can parallel and match it."

Source: Ibn Jubayr, Spanish Muslim traveler and geographer, 1184 CE

"Cordoba after the Arab conquest of the Iberian Peninsula became the ultimate goal of the land, headquarters of the army, mother of all towns, seat of the virtuous and pious, and abode of the people of intellect and learning...The explanation of the superiority of the Cordobans over all others past and present lies in the fact that the city's climate is one of research and investigation in the variety of sciences and literature."

Source: Al-Hijari, Andalusian (Spanish Muslim of Arab origin) historian, from his writings, 1188

21. The statements of both Muslim authors share the same goal of?

(A) encouraging non-Muslims to come to Arabia and convert to Islam

(B) encouraging loyal Muslims to join the Crusades

(C) spreading the ideals of Islam across the entire region

(D) portraying the cities of Islam as the most virtuous places on Earth

22. Ibn Jubayr issued the writingsprimarily in order to

(A) show from a Muslim perspective the splendors of an Islamic city

(B) gain support from Muslims in far off lands like India and China

(C) protect Muslim interests in the Middle East

(D) support the expansionist policies of the Umayyad Dynasty

23. Al-Hijari's writing best reflects which of the following developments in the spread of Islam?

(A) the expansionist policies of the Umayyad

(B) the philosophical and scholarly atmosphere of Islamic Spain

(C) the contrast between Islamic and Christian cities

(D) the demand of Muslims to join the Crusades to defeat the Christians

Questions 24 - 26 refer to the late 20th century photograph by Stuart Franklin.

24. Conditions like those shown in the image contributed most directly to which of the following?

(A) an increase in the spread of U.S. military bases in places like Okinawa

(B) an increase in the rise of Communist party operations

(C) a decline in the use of American troops to spread democracy

(D) The decline of Communist operations in East Asia

25. The conditions shown in the image depict which of the following trends in the mid 20th century?

(A) the growing division between rich and poor in society

(B) the rise of a counter-culture movement

(C) the use of military force as a means of promoting democracy

(D) the rise of the United Nations

26. Advocates for individuals such as those shown in the image would have most likely agreed with which of the following perspectives?

(A) the establishment of the United Nations by the Atlantic Charter

(B) Capitalism, free of all government regulation, would improve society

(C) collective security would lead to too many conflicts

(D) Democracy was at risk, and should be protected at all costs

Questions 27 - 29 refer to the excerpt below.

" Papal indulgences for the building of St. Peter's are circulating under your most distinguished name, and as regards them, I do not bring accusation against the outcries of the preachers, which I have not heard, so much as I grieve over the wholly false impressions which the people have conceived from them; to wit, -- the unhappy souls believe that if they have purchased letters of indulgence they are sure of their salvation; again, that so soon as they cast their contributions into the money-box, souls fly out of purgatory; furthermore, that these graces [i.e., the graces conferred in the indulgences] are so great that there is no sin too great to be absolved, even, as they say -- though the thing is impossible -- if one had violated the Mother of God; again, that a man is free, through these indulgences, from all penalty and guilt."

Martin Luther, Letter to Archbishop of Mainz, 1517

27. Which of the following aspects of Luther's description expresses a major change in Europeans' views of Catholicism?

(A) The idea that indulgences can be sold but only by the Pope

(B) The idea that religion has a role to play in ensuring economic codes

(C) The idea that salvation can be reached by following the sacraments

(D) The idea that the Pope and Catholic church were corrupt and in need of a religious reformation

28. Luther's ideas are most directly a reaction to the

(A) the increasing sentiments that the Catholic church was interested

in money more than the gospel

(B) growing backlash of Catholics against the Protestants

(C) the increase in demand for new Christian converts

(D) the persecution of religious minorities during the Inquisition

29. Luther's position regarding the papal sale of indulgences was most strongly supported by which of the following?

(A) Italian merchants

(B) members of the Spanish Crown

(C) the Catholic Church

(D) Calvinists

Questions 30-33 refer to the excerpt below.

"If a woman bring about the death of her husband for the sake of another man, they shall impale her.

If a son strike his father, they shall cut off his fingers. If a man destroy the eye of another man, they shall destroy his eye. If one break a man’s bone, they shall break his bone... If a man knock out a tooth of a man of his own rank, they shall knock out his tooth... If he be a freeman, he (the physician) shall receive five shekels... If a physician operate on a man for a severe wound with a bronze lancet [surgical knife] and cause the man’s death; or open an abscess (in the eye) of a man with a bronze lancet and destroy the man’s eye, they shall cut off his fingers..."

Hammurabi, Law Code (translated), 1,800 B.C.E. 

30. The issues expressed by Hammurabi were a response to the?

(A) controversy regarding legal support for various minority religions

(B) the need to develop a uniform and public law code to unify his people

(C) debate over the treatment of women

(D) the dispute over monotheism

31. The ideas expressed in Hammurabi's law code most strongly influenced which trend in ancient Mesopotmia?

(A) The establishment of legal codes

(B) the equal treatment of women throughout Mesopotamia

(C) the refusal to protect minority groups such as women and slaves

(D) The development of free public health care

32. The excerpt would be most useful to historians as a source of information about which of the following

(A) The role that law codes play in ancient Mesopotamian people's lives

(B) the relative equality of all citizens under the protection of the law in ancient Mesopotamia

(C) the harsh realities of life for women under a patriarchal society

(D) The interaction between Mesopotamian rulers and his people

33. Which of the following developments demonstrated the strongest continuity with the idea expressed in the decree?

(A) Modern democracies are based on the rule of law

(B) The idea of equal pay for women

(C) The need for no cruel and unusual punishment

(D) The separation of religious and political law codes

Questions 34 - 36 refer to the diagram below.

Courtesy of Wikicommons

34. The import of Old World crops to the New World transformed American society mostly by?

(A) diversifying diets and thereby triggering a population increase

(B) encouraging the privatization of lands

(C) improving relations between Europeans and native inhabitants

(D) encouraging warfare between European powers

35. The patterns depicted in the diagram most directly foreshadowed which of the following developments?

(A) the spread of corn from South America to North America

(B) the gradual transition from a feudal to capitalist society

(C) The population decline of Native Americans

(D) the emergence of racially mixed populations in the Americas

36. The trends portrayed in the diagrammost closely corresponds with which of

the following major historical developments in the Atlantic world?

(A) The rise of the Triangle Trade

(B) The Manila Galleons

(C) The growth of imperialism in Africa

(D) The Plague

Questions 37 - 39 refer to the map below.The Spread of Islam 622-750 C.E.

37. The pattern of Islamic conversions after 622 C.E. were linked most directly by which of the following factors?

(A) the large size of the Mediterranean Sea in relation to Arabia

(B) the expansionist policies of the first four caliphs and the Umayyad Dynasty

(C) the production of luxury goods to be sold in Asia along the Silk Road

(D) The Byzantine government's inability to impose strict control in its territories

38. The difference in conversion patterns from the first four caliphs to

that of the Umayyads had which of the following effects?

(A) the promotion of science, literature, and philosophy in Spain

(B) a decrease in the settlements along the Mediterranean

(C) the decline in the importance of Mecca as a holy city

(D) a major increase in trade with China

39. The difference in trade patterns between the land and sea routes best explains the

(A) significance of understanding how to navigate the Monsoon winds

(B) development of religious differences between Africans, and Persians

(C) spread of the Bubonic Plague

(D) growth of religious tensions between Christians and Muslims

Questions 40 - 42 refer to the excerpt below.

"... Upon this a question arises: whether it is better to be loved than feared or feared than loved? It may be answered that one should wish to be both, but, because it is difficult to unite them in one person, it is much safer to be feared than loved,... men have less scruple in offending one who is beloved than one who is feared, for love is preserved by the link of obligation which, owing to the baseness of men, is broken at every opportunity for their advantage; but fear preserved you by a dread of punishment which never fails... Nevertheless a prince ought to inspire fear in such a way that, if he does not win love, he avoids hatred; because he can endure very well being feared whilst he is not hated, which will always be as long as he abstains from the property of his citizens and subjects and from their women.

Niccolo Machiavelli, The Prince, 1513

40. Which of the following groups would have been most likely to support Machiavelli's views expressed in the excerpt?

(A) Conservative Democrats

(B) an elected Senator

(C) Absolute monarchs

(D) the Pope

41. Which of the following most directly undermines Machiavelli's claims?

(A) the appeal of free and democratic elections in various countries

(B) the public support for Nazism gained by Adolf Hitler

(C) the rule of Joseph Stalin in the Soviet Union

(D) a majority of Enlightenment philosophes were deists

42. In the 20th century, the views expressed by Machiavelli most directly contributed to

(A) increased political tension during the Cold War

(B) the use of violence to achieve colonial independence

(C) the rise of fascism and dictatorshipthroughout Europe

(D) the rising tide of nationalism that swept across the Middle East after WWI

Questions 43 - 44 refer to the excerpt below.

"The Persians do not eat the flesh of cows and calves, but mutton to a vast extent and horseflesh, which is the most esteemed and by the nobles... Their garb is a long garment, different from that of the Turks: they tie shawls round their waists, and almost all of them go clothed in cotton stuffs of various colours in imitation of the king. Their chief food is rice with meat, and they do not use such variety, nor dainties as in these countries [of Europe]: and they are frugal and satisfied with little food... Almost all of them drink wine: they sit and eat on the ground... on rich carpets... On the street side they have no windows, so that their women should not be seen: and thus the streets are not attractive, nor is the city fine... There are some of them, who profess to be philosophers and mathematicians, almost all of them to be poets: and they continually have books in their hands. They have many large mosques, where they go to say their prayers, and they allow any nation whatsoever to enter them..."

Father Simon, Report on Persia, c. 1604 C.E.

43. The excerpt would be most useful to historians as a source of information about which of the following?

(A) the harsh realities of living under such a repressive regime

(B) the role that food plays in the everyday lives of Safavid Muslims

(C) the conflicts between Europeans and Persians in the early modern era

(D) the cultural customs of the Persians under the Safavid Dynasty

44. Which of the following was a major contrast between the customs of the Persians and those of the Turks?

(A) the Ottomans had cross-cultural interactions with Europeans

(B) the Ottomans were considered one of the main Islamic gunpowder empires

(C) the Persians practiced Shi'a Islam, which venerated the lineage of Ali

(D) the Persians promoted law and education

Questions 45 - 47 refer to the excerpt below.

" Govern the state by correctness;

Deploy the army by deception;

Acquire the empire by taking no action (wushi).

How do I know this is so?

By this.

The more prohibitions there are in the world, The poorer are the people.

The more sharp weapons the people have, The more disorder is fomented in the family and state.

The more adroit and clever men are, The more deceptive things are brought forth.

The more laws and ordinances are promulgated, The more thieves and robbers there are.

."

Laozi, Daodejing, c. 500 B.C.E

45. The Daoist account most directly illustrated the debates about which of

the following issues in classical China?

(A) The limits of the government

(B) The proper role of government

(C) The social tensions between farmers and landlords

(D) The clash between military and government officials

46. The attitudes of prohibitions described by Laozi is most consistent with which of the following developments in the 20th century?

(A) The Great Leap Forward and Cultural Revolution

(B) the Soviet Union during the Stalin era

(C) the independence of India

(D) the transition of a communist country to capitalism

47. Efforts by supporters of Laozi to gain official promotion by the Hangovernment ultimately failed because

(A) the Han rulers favored laws more based on harmony with nature

(B) Daoists weren't interested in gaining s converts

(C) the popularity of other philosophies like Confucianism, and Buddhism among popular and elite forces

(D) The Daoists were always plagued with internal power struggles

Questions 48 - 50 refer to the excerpt below.

“When the war in Kalinga was over, and all the people were conquered, he [Ashoka] felt inside him a great crisis, a stirring for meaning and a remorse. Ashoka goes on a pilgrimage seeking a guru, a teacher. And by the riverbank he met a Buddhist monk and the monk told him to sit beneath the Bodhi tree where the Buddha had found Enlightenment. And there the power of ideas and the power of the state came together in a uniquely Indian way. [It was] a rejection of the path of violence [and] of a whole way of understanding history. While he was here, Ashoka gave rich gifts to the poor. He consulted with local communities about proper governance, about good conduct...forming in his mind now as a political order the sort of which had never been conceived of before in the history of the world."

- Michael Wood, PBS's Story of India (2009)

48. The Battle of Kalinga was significant in the history of India because it

(A) was the first major battle against a neighboring state

(B) led to the decline of trade

(C) the most violent conflict up to that point

(D) it was a turning point in the promotion of Buddhism

49. The pilgrimage taken by Ashoka most directly reflected?

(A) a desire to expand territory

(B) the desire to put philosophical and political theory into practice

(C) social tensions between the Hindus and Buddhists

(D) to spread Buddhism across Asia

50. Which of the following evidence would best support Wood's argument in the excerpt?

(A) Military documents from all of the major civilizations up to that point

(B) Testimony from government officials that Ashoka was influential on modern politics

(C) Narratives from the lives of Indians who and designed the Indian national flag

(D) Statistical data revealing the number of Buddhists in India before and after Ashoka's pilgrimage

Questions 51 - 53 refer to the political cartoon below.

Courtesy of Columbia University

51. The poster was intended to

(A) persuade man to read more diverse types of literature

(B) promote the ideals of the Chinese communist revolution

(C) advocate for the promotion of men and women in government positions

(D) convince peasants to join the Cultural Revolution

52. The poster most directly reflects the

(A) emergence of China as an industrial power

(B) mobilization of all social classes in support of the Cultural Revolution and it's omnipotent leader

(C) the victory over the defeat of Japan after WWII

(D) the diversity of opinions during the Cultural Revolution

53. Which of the following represents a later example of a change in policy from the campaign highlighted in the poster?

(A) The restrictions on the internet

(B) The government's decision to limit religious freedom

(C) China's liberalization of the economy after Deng Xiaoping opened the economy to western business

(D) China's transition to a manufacturing economy

Questions 54 - 55 refer to the excerpt below.

" It is true that South Africa was often brought to the brink of destruction because of differences... Since we have achieved our freedom, there can only be one division amongst us: between those who cherish democracy and those who do not! As freedom loving people, we want to see our country prosper and provide basic services to all. For our freedom can never be complete or our democracy stable unless the basic needs of our people are met...As we rebuild our country, we should remain vigilant against the enemies of development and democracy, even if they come from within our own ranks. Violence will not bring us closer to our objectives. All of us should ask ourselves the question: Have I done everything in my power to bring about lasting peace and prosperity in my city and my country? ...

Nelson Mandela, On Reconciliation, 04/16/1999

54. The excerpt best reflects an effort by Mandela to encourage?

(A) the landowning whites to give up some of their land

(B) promote unity and integration in a post-Apartheid South Africa

(C) the enactment of socialist programs for all social services so as to make them dependent upon the government

55. The ideas expressed by Mandela in the excerpt were most similar to those of which twentieth century world leader?

(A) Mikhail Gorbachev

(B) Ho Chi Minh

(C) John F. Kennedy

(D) Mahandas Gandhi

(D) the construction of a democratic political system based on the two party system

END OF PART AIF YOU FINISH BEFORE TIME IS CALLED, YOU MAY CHECK YOUR WORK ON PART A. DO NOT GO ON TO PART B UNTIL YOU ARE

TOLD TO DO SO.

AP__________________________

Sample AP World History2017 Free-Response Questions__________________________

Directions: Read each passage carefully. Answer on a separate paper using complete sentences- bullet points are not acceptable. 4 questions; 50 minutes.

1) Answer a, b, and ca) Briefly explain ONE important similarity between the Bantu and Meso-American migrations in populating their respective continents prior to 500 B.C.

b) Briefly explain ONE important difference between the Bantu and Meso-American migrations in populating their respective continents prior to 500 B.C.

c) Briefly explain ONE way in which the difference you indicated in b contributed to a difference in the development of pre-Columbian civilizations in the Americas or traditional African civilizations.

Use the diagram below to answer all parts of the question that follows.

"Woman is a violent and uncontrolled animal... If you allow them to achieve complete equality with men, do you think they will be easier to live with? Not at all. Once they have achieved equality, they will be your masters."

Source: Cato the Elder, Speech in the Roman Senate (195 B.C.)

"Why should we pay taxes when we do not share in the offices, honours, military commands, nor, in short, the government, for which you men fight between yourselves, with such harmful results?"

Source: Public Speech given by group of Roman women

2. a) Describe ONE important difference between the views of women and citizenship expressed in the two passages.

b) For EACH of the passages, identify and explain ONE factor (such as a historical development, an intellectual or philosophical trend, or a religious belief) that likely informed the view of women and citizenship expressed in the passage.

Answer all parts of the question that follows.

3.Many historians argue that the Mongol conquest of Baghdad in 1258 was a significant turning point in world history.

a) Identify ONE specific piece of evidence that supports the argument, and explain how the piece supports the contention.

b) Identify ONE specific piece of evidence that undermines the argument, and explain how it undermines the contention.

c) Briefly explain how ONE person, event, or development from the period 500-1500 could be used to support the argument.

Use the passage below to answer all parts of the question that follows.

"... No one ought to harm another in his life, health, liberty, or possessions... ; for without this the law could not have that which is absolutely necessary to its being a law, the consent of the society, over whom nobody can have a power to make laws but by their own consent and by authority received from them….: They must not raise taxes on the property of the people without the consent of the people given by themselves or their deputies.... When any one, or more, shall take upon them to make laws whom the people have not appointed so to do, they make laws without authority, which the people are not therefore bound to obey; by which means they come again to be out of subjection, and may constitute to themselves a new legislative, as they think best, being in full liberty to resist the force of those who, without authority, would impose anything upon them…."

John Locke: Two Treatises of Government (1689)

1. a) Briefly explain ONE implication for public policy of Locke's view on society.

b) Identify and explain ONE factor (such as a historical development, an intellectual or philosophical trend, or a religious belief) that likely informed the view of authority and society expressed in the passage.

c) Briefly explain how ONE person, event, or development from the period 1750-1914 that is not explicitly mentioned in the excerpts could be used to support Locke's interpretation.

END OF SECTION 1

END OF SECTION 1IF YOU FINISH BEFORE TIME IS CALLED, YOU MAY CHECK YOUR WORK ON PART A. DO NOT GO ON TO PART B UNTIL YOU ARE

TOLD TO DO SO.

WORLD HISTORY

SECTION II

Total Time-- 1 hour, 30 minutes

Question 1 (Document-Based Question)

Suggested reading and writing time: 55 minutes

It is suggested that you spend 15 minutes reading the documents and 40 minutes writing your response.

Note: You may begin writing your response before the reading period is over.

Directions: Question 1 is based on the accompanying documents. The documents have been edited for the purpose of this exercise.

In your response you should do the following:

Thesis: Present a thesis that makes a historically defensible claim and responds to all parts of the question. The thesis must consist of one or more sentences located in one place, either in the introduction or the conclusion.

Argument Development: Develop and support a cohesive argument that recognizes and accounts for historical complexity illustrating relationships among historical evidence such as contradiction, collaboration, and/or qualification.

Use of the Documents: Explain the significance of the author's point of view, author's purpose, historical context, and/or audience for at least four documents.

Contextualization: Situate the argument by explaining the broader historical events, developments, or processes immediately relevant to the question.

Outside Evidence: Provide an example or additional piece of specific evidence beyond those found in the documents to support or qualify the document.

Synthesis: Extend the argument by explaining the connections between the argument and ONE of the following.

A development in a different historical period, situation, era, or geographical area. A course theme and/ or approach to history that is not the focus of the essay (such as political,

economic, social, or intellectual history) A different discipline or field of inquiry (such as economics, government and politics, art history,

or anthropology).

1. Explain the causes of the rise of the worker's rights movement in Europe during the period 1750-1914.

Document 1

Factory Wages in Lancashire, England in 1830Age of Worker Male Wages Female Wages

under 11 2s 3d. 2s. 4d.11 - 16 4s. 1d. 4s. 3d.17 - 21 10s. 2d. 7s. 3d.22 - 26 17s. 2d. 8s. 5d.27 - 31 20s. 4d. 8s. 7d.32 - 36 22s. 8d. 8s. 9d.37 - 41 21s. 7d. 9s. 8d.42 - 46 20s. 3d. 9s. 3d.47 - 51 16s. 7d. 8s. 10d.52 - 56 16s. 4d. 8s. 4d.57 - 61 13s. 6d. 6s. 4d.

Source: Spartacus Educational.com

Document 2

Betty Harris, age 37: I was married at 23, and went into a colliery when I was married. I used to weave when about 12 years old; can neither read nor write. I work for Andrew Knowles, of Little Bolton (Lancs), and make sometimes 7s a week, sometimes not so much. I am a drawer, and work from 6 in the morning to 6 at night. Stop about an hour at noon to eat my dinner; have bread and butter for dinner; I get no drink. I have two children, but they are too young to work. I worked at drawing when I was in the family way. I know a woman who has gone home and washed herself, taken to her bed, delivered of a child, and gone to work again under the week.

I have a belt round my waist, and a chain passing between my legs, and I go on my hands and feet. The road is very steep, and we have to hold by a rope; and when there is no rope, by anything we can catch hold of. There are six women and about six boys and girls in the pit I work in; it is very hard work for a woman. The pit is very wet where I work, and the water comes over our clog-tops always, and I have seen it up to my thighs; it rains in at the roof terribly. My clothes are wet through almost all day long. I never was ill in my life, but when I was lying in.

My cousin looks after my children in the day time. I am very tired when I get home at night; I fall asleep sometimes before I get washed. I am not so strong as I was, and cannot stand my work so well as I used to. I have drawn till I have bathe skin off me; the belt and chain is worse when we are in the family way. My feller (husband) has beaten me many a times for not being ready. I were not used to it at first, and he had little patience...

- Testimony from a woman worker to the British Government (1842)

Document 3

Q: 'What was the effect of this state of the work-places upon the habits of the workmen?

A: It had a very depressing effect on the energies;...The natural effect of the depression

was, that we had recourse to drink as a stimulant. We went into the shop at six o'clock

in the morning; but at seven o'clock, when orders for the breakfast were called for, gin

was brought in, and the common allowance was half-a quart. The younger hands did

not begin with gin.

Q:'Was gin the first thing taken before any solid food was taken?--

A: Yes, and the breakfast was very light; those who took gin generally took only half-a

pint of tea and half a two penny loaf as breakfast.

Q:'When again was liquor brought in?-

A:At eleven o'clock. 'What was taken then?-Some took beer, some took gin again. In a

general way, they took a pint of porter at eleven o'clock. It was seldom the men took

more than the half-quart of gin.

Q: 'When again was liquor brought in ?

A:-At three o'clock, when some took beer and some gin, just the same as in the

morning. At five o'clock the beer and gin came in again, and was usually taken in the

same quantities. At seven o'clock the shop was closed.

Q:'What were the wages they received?-

A:Sixpence per hour, which, at the full work, made 6s. a-day, or 36s. a-week. Q:'Did

they make any reserves from this amount of wages ?

A:No; very few had anything for themselves at the end of the week.

Q:'How much of the habit of drinking was produced by the state of the workplace?

-A:I should say the greater part of it; because when men work by themselves, or only

two or three together, in cooler and less close places, there is scarcely any drinking

between times. Nearly all this drinking proceeds from the large shops, where the men

are crowded together in close rooms...

- From Sir Edwin Chadwick's official report on labor conditions- this excerpt is an interview

with a factory tailor (1842)

Document 4

Source: Political Cartoon on labor during the Industrial Revolution.

Document 5

Mr. -------- remarked that nothing could be so beneficial to a country as manufacture. 'You see these children, sir,' said he. 'In most parts of England poor children are a burden to their parents and to the parish; here the parish, which would else have to support them, is rid of all expense; they get their bread almost as soon as they can run about, and by the time they are seven or eight years old bring in money. There is no idleness among us: they come at five in the morning; we allow them half an hour for breakfast, and an hour for dinner; they leave work at six, and another set relieves them for the night; the wheels never stand still.'

I was looking, while he spoke, at the unnatural dexterity with which the fingers of these little creatures were playing in the machinery, half giddy myself with the noise and the endless motion; and when he told me there was no rest in these walls, day or night, I thought that if Dante had peopled one of his hells with children, here was a scene worthy to have supplied him with new images of torment.

'These children then,' said I, 'have no time to receive instruction.' 'That, sir,' he replied 'is the evil which we have found. Girls are employed here from the age you see them till they marry, and then they know nothing about domestic work, not even how to mend a stocking or boil a potato. But we are remedying this now, and send the children to school for an hour after they have done work.' I asked if so much confinement did not injure their health. 'No' he replied, 'they are as healthy as any children in the world could be. To be sure, many of them as they grew up went off in consumptions, but consumption was the disease of the English. ...''We are well off for hands in Manchester,' said Mr. ------; 'manufacturers are favourable to population, the poor are not afraid of having a family here, the parishes therefore have always plenty to apprentice, and we take them as fast as they can supply us. In new manufacturing towns they find it difficult to get a supply. Their only method is to send people round the country to get children from their parents. Women usually undertake this business; they promise the parents to provide for the children; one party is glad to be eased of a burden; and it answers well to the other to find the young ones in food, lodging and clothes, and receive their wages.' 'But if these children should be ill-used', said I. 'Sir,' he replied, 'it never can be the interest of the women to use them ill, nor of the manufacturers to permit it.'It would have been in vain to argue had I been disposed to it. Mr. ------- was a man of humane and kindly nature, who would not himself use any thing cruelly, and judged of others by his own feelings. I thought of the cities in Arabian romance, where all the inhabitants were enchanted: here Commerce is the Queen witch, and I had no talisman strong enough to disenchant those who were daily drinking of the golden cup of her charms.

-A skeptical Robert Southey interviews a proud factory owner. (1807)

Document 6As to the conclusions I have come to from the working of my mill for 11 instead of 12 hours each

day, as previously, I am quite satisfied that both as much yarn and cloth may be produced at

quite as low a cost in 11 as in 12 hours. It is my intention to make a further reduction to 10½

hours, without the slightest fear of suffering loss. I find the hands work with greater energy and

spirit; they are more cheerful, and happy. All the arguments I have heard in favour of long time

appear based on an arithmetical question - if 11 produce so much, what will 12 or 15 hours

produce? This is correct, [for] the steam-engine, but try this on the horse, and you will find he

cannot compete with the engine, as he requires time to rest and feed.

There is more bad work made the last one or two hours of the day than the whole of the first

nine of ten hours. About 20 years ago, we had many orders for a style of goods. We had about

30 young women in our Manchester warehouse; I requested that they would work [instead of

11] 12 hours. At the end of the week, I found they had not a mere trifle more work done' but,

supposing there was some incidental cause for this, I requested they would work 13 hours the

following week, at the end of which they had produced less instead of more work. The

overlooker invited me to be in the room with them the last hour of the day. They were exhausted

and making bad work and little of it. I therefore reduced their time two hours, as before. Since

that time I have been an advocate for shorter hours of labour.

A Wise Factory Owner: Parliamentary Papers (1845)

Document 7

The history of all hitherto existing society is the history of class struggles... The modern

bourgeois society that has sprouted from the ruins of feudal society, has not done away with

class antagonisms...Society as a whole is more and more splitting up into two great hostile

camps, into two great classes directly facing each other bourgeoisie (factory owners) and

proletariat (factory workers /wage slaves). . .

The Communists disdain to conceal their views and aims. They openly declare that their ends

can be attained only by the forcible overthrow of all existing social conditions. Let the ruling

classes tremble at a Communistic revolution. The proletarians have nothing to lose but their

chains. They have a world to win. Working men of all countries, unite!

Karl Marx: Communist Manifesto 1848

SECTION II

Total Time- 1 hour, 30 minutes

Question 2 or Question 3

Suggested writing time: 35 minutes

Directions: Choose EITHER question 2 or question 3.

In your response you should do the following.

Thesis: Present a thesis that makes a historically defensible claim and responds to all parts of the question. The thesis must consist of one or more sentences located in one place, either in the introduction or conclusion.

Application of Historical Thinking Skills: Develop and support an argument that applies historical thinking skills as directed by the question.

Supporting the Arguments with Evidence: Utilizes specific examples of evidence to fully and effectively substantiate the stated thesis or relevant argument.

Synthesis: Extend the argument by explaining the connections between the argument and ONE of the following.

A development in a different historical period, situation, era, or geographical area. A course theme and/or approach to history that is not the focus of the essay (such as political,

economic, social, cultural, or intellectual history). A different discipline or field of inquiry (such as economics, government and politics, art history,

or anthropology).

3) Evaluate the extent to which the Chinese Revolution in 1949 marked a turning point in the triumph of traditional Communism in world history, analyzing what changed and what stayed the same from the period before the revolution to the period after it. (Historical Thinking Skill: Periodization)

or

2.) Evaluate the extent to which the achievement of Indian independence in 1947 was a significant turning point in the in the world's history of decolonization.

In the development of your argument, explain what changed and what stayed the same from the period immediately before the decolonization of India to the period immediately following it.. (Historical Thinking Skill: Periodization)

STOP---END OF EXAM

AP__________________________

Sample AP World History

2017 Multiple-Choice Answers

__________________________

Answers to Multiple-Choice Questions

1. (D) traditions and practices of Islam

2. (C) the rise and spread of Islam

3. (A) The development of the Silk Road

4. (A) Active encouragement of export and manufacturing for the world economy

5. (C) manufacture and export

6. (B) the environment in urban areas has become polluted and hazardous

7. (A) The convergence of world markets in the age of globalization

8. (C) patriarchy was viewed as legitimate

9. (A) The 19th Amendment to the U.S. Constitution, granting women the right to vote

10. (C) the failure by many liberals to grant women full equality and citizenship

11. (D) political and social acceptance of torture as a means of discipline and punishment for

suspected criminals

12. (D) The ideas of the Enlightenment

13. (A) The increase in the rights of the accused like innocent until proven guilty

14. (C) the Arab territories of Transjordan, Palestine, Syria, and Lebanon were divided among

Europeans

15. (D) a mandate system under European influence

16. (B) Berlin Conference

17. (B) 14 Points Plan

18. (A) lauds the possible benefits of the slave trade

19. (C) encourage the improved treatment of the slaves

20. (B) the success of the Haitian Revolution

21. (D) portraying the cities of Islam as the most virtuous places on Earth

22. (D) support the expansionist policies of the Umayyad Dynasty

23. (B) the philosophical and scholarly atmosphere of Islamic Spain

24. (A) an increase in the spread of U.S. military bases in places like Okinawa

25. (C) the use of military force as a means of promoting democracy

26. (D) Democracy was at risk, and should be protected at all costs

27. (D) The idea that the Pope and Catholic church were corrupt and in need of a religious

reformation

28. (A) the increasing sentiments that the Catholic church was interested in money more than

the gospel

29. (D) Calvinists

30. (B) the need to develop a uniform and public law code to unify his people

31. (A) The establishment of legal codes

32. (B) the relative equality of all citizens under the protection of the law in Mesopotamia

33. (A) Modern democracies are based on the rule of law

34. (A) diversifying diets and thereby triggering a population increase

35. (C) The population decline of Native Americans

36. (A) The rise of the Triangle Trade

37. (B) the expansionist policies of the first four caliphs and the Umayyad Dynasty

38. (A) the promotion of science, literature, and philosophy in Spain

39. (A) significance of understanding how to navigate the Monsoon winds

40. (C) Absolute monarchs

41. (A) the appeal of free and democratic elections in various countries

42. (C) the rise of fascism and dictatorship throughout Europe

43. (D) the cultural customs of the Persians under the Safavid Dynasty

44. (C) the Persians practiced Shi'a Islam, which venerated the lineage of Ali

45. (B) The proper role of government

46. (A) The Great Leap Forward and Cultural Revolution

47. (C) the popularity of other philosophies like Confucianism, and Buddhism among popular and

elite forces

48. (D) it was a turning point in the promotion of Buddhism

49. (B) the desire to put philosophical and political theory into practice

50. (A) Military documents from all of the major civilizations up to that point

51. (B) promote the ideals of the Chinese communist revolution

52. (B) mobilization of all social classes in support of the Cultural Revolution and it's omnipotent

leader

53. (C) China's liberalization of the economy after Deng Xiaoping opened the economy to

western business

54. (B) promote unity and integration in a post-Apartheid South Africa

55. (D) Mahandas Gandhi

AP__________________________

Sample AP World History2017 Free-Response Answers

__________________________

Question 1

This question is about similarities in migration patterns between Bantu and American populations during the ancient era.

a) The biggest geographic similarity is that they both began in the northwest before spreading south. The Americans began in Russia, then spread from Alaska to the southern tip of Argentina- Cape Horn. The Bantus spread from the Niger region to the southern tip of Africa- the cape of Good Hope.

The linguistic influence was similar in both cases. Bantu migrations helped spread the various languages and dialects throughout Africa. Similarly, the American migrations helped spread the indigenous languages from north to south of the continent.

The agricultural impact was also shared in common. The Bantus helped spread the technology of plowing and various cultivation techniques throughout Africa. In a similar manner, the American migrations eventually led to established civilizations like the Olmecs, Toltecs, Aztec, Inca, and Mayas.

b) The main difference was their place of origins. The Americans originally came from the Mongolia/ Russia area before they crossed the land bridge across Alaska into the Americas. However, the Bantus began in Africa before spreading throughout the rest of Africa. The Americans can be traced back to the Mongoloid race whereas the Bantu are distinctly African.

c) The evidence of Clovis arrowheads is one difference resulting from the migrations into the Americas. As the American nomads began chasing big game like deer and buffalo across the continent, they developed arrowhead technology to hunt animals. That accounts for the existence of native American arrowheads scattered across the continent.

The Bantu societies never fully improved their agricultural methods. As a result, mostly small scale farming with traditional technology and stone tools were used across Africa until the time of imperialism.

Question 2

The question is based on comparing two passages on women and citizenship during the age of the Roman empire.

a) The main difference between the two passages is that the first author doesn't believe in women's equality whereas the second passage demands full citizenship on equal terms. The first passage see women's equality as a form of slavery over men, but the second passage views women's equality as true freedom. Each author sees the other sex as the chaotic one causing problems in society.

b)The first passage was written from a patriarchal perspective. The man clearly expresses how he feels that women's equality means male slavery, which is why he wants to dominate women in society.

The second passage expresses a belief in women's rights and equality. Perhaps, she was influenced by democratic ideals. Maybe she was impacted by the Roman democracy, which granted freedoms to the plebian (lower) classes. She expresses an early fight for women's rights, that wouldn't really be seen until the 19th century.

Question 3

This question asks you to explain why the Mongol conquest of Baghdad in 1258 was a turning point in history.

a) The biggest effect of the Mongol conquest of Baghdad was the fall of the Abassid Dynasty. The Abassids were a Muslim dynasty who ruled over the Muslim world from 750-1258 C.E. They had been the largest and most powerful empire in the region until they were crushed by the Mongols. The Abassid caliph was murdered by the invaders as he was trampled by horses. The fall of Baghdad led to a decline in literature and education in the city. The city was a center for learning and scholarship before the Mongols. After the Mongol invasion, the city was looted and books were burned. Much of the knowledge had been lost due to the Mongol invasion. The next major ruling Muslim empire would be ruled by the Ottoman Turks, no longer by the Abassid Arabs.

b) One piece of evidence that contradicts the statements in a is that Islam and Islamic empires still continued long after the fall of Baghdad. It's not like the fall of Baghdad, led to the disappearance of Islam. On the contrary, Islam spread and flourished under three very strong empires during the modern era: Ottoman Turks, Persian Safavids, and Indian Mughals. You might think that the fall of the greatest Islamic empire had a devastating effect on the religion, but that wasn't the case. Islam continue to thrive long after the fall of Baghdad. Plus, the holiest place in Islam will always be in Mecca, regardless of where the caliph lives.

c) The Abassid caliph Harun al-Rashid ruled the empire at its high point. He helped make the Abassids the greatest Islamic empire of its time. The Abassids are known as the golden age of Islam. Harun al Rashid funded the Dar al Hikma in Baghdad, or House of Knowledge. This was a scholarly center where education and science flourished. Once the Mongols sacked the city, all the knowledge and hard work of Harun Al Rashid had been laid to waste.

Another possible development includes the rise of the Mongol empire after Gengis Khan. After the Mongols came to power, they decided to spread their empire as vast as they could extend it. Without the rise of the Mongols, the conquest of Baghdad never would have been possible.

Question 4

This question is an excerpt from John Locke's Two Treatises of Government.

a) One implication is that the people could overthrow their government in a revolution and establish a democracy. Locke's ideas were actually put into practice in all three democratic revolutions- American, French, and Haitian. Locke said, the government can't rule without the "consent of the people" or else they were justified in overthrowing the government and "may constitute themselves a new legislature."

b) Most likely Locke was influenced by the Scientific Revolution. His ideas of natural laws, and rights were influenced by the natural laws of science. Locke just applied those scientific laws to politics, helping to found the field of political science. He believed that people had inalienable rights and nobody had the right "to harm another in his life, health, liberty, or possessions." Scientists like Copernicus, Galileo, Bacon and Newton all believed in the natural laws of the universe. Similarly, Locke believed in the natural rights of citizens in a democracy.

c) The entire era of democratic revolutions was heavily influenced by Locke. The revolutions in America, France, and Haiti were all influenced by Locke. They all used his language in their founding political documents for their new democracies. In America, Thomas Jefferson write about the rights of "life, liberty, and the pursuit of happiness," heavily borrowed from Locke's writings. In France, they established the rights for all French citizens based on Locke's ideas. Haiti was founded on the principles that no one had the right to take away their liberty or liberty.

SECTION II, PART A: DOCUMENT BASED QUESTION (EXPLAINED)

Document 1

This document is based on a chart of factory wages in a British city in 1830 C.E. The chart ranges from ages 11-61, and gives the brackets for male and female wages. It clearly shows how at a young age females actually made more money than males. However, after age 17 males were paid higher wages than females. It shows how wages were not set at a minimum wage as there were no government labor laws in 1830. It also shows how kids and females were being used as factory labor. The chart says a lot because it shows how worker's were not paid a uniform, objective rate for the job, but one that was subjectively chosen by the employer. That explains why women and kids were usually paid less than young men. It is unclear if these factories hired a higher number of male or female workers.

Document 2

This is a fascinating document because it is testimony to the British government from a woman miner during the industrial revolution. She describes her difficult working conditions. She started working at 12 years old. She worked a very difficult, and dirty job with long hours for very little pay. She works a 12 hour shift every day with a one hour break for lunch. It shows how difficult the working conditions were, and how women workers were mistreated during the Industrial Revolution. At the end, she mentions how her husband also abuses her at times. Definitely the working conditions add stress on her family life as well, including the husband and children.

Document 3

In this classic government report from the industrial revolution era, the author is reporting on the horrible working conditions. He interviews a worker in this passage, and asks him about all of the alcohol he consumes all day at work. The worker admits that they usually drank before work, because they were depressed. The employer would provide the alcohol as fuel to keep the worker's motivated. This document really demonstrates how different working conditions were in those days in industrialized countries today.

Document 4

Here, the political cartoon depicts a group of people pulling a big, fat man smoking a cigar on a wagon whose caption reads "supported by child labor." There are children and even girls pulling the wagon with the help of an adult. The man, who represents a capitalist factory owner, is just sitting there while the kids do all of the work. The cartoon is criticizing capitalism and the factory bosses by saying that they are fat cats who do nothing but smoke cigars and drink alcohol, living off the hard work of others- in this case children. The cartoon would was probably written by a communist sympathizer or at least a labor reform advocate.

Document 5

This document is a personal letter about an interview with a factory owner explaining all of the benefits of child labor for not only himself but also the children and their families. The interviewer is quite skeptical and dubious about having children work in factories for such long hours under such harsh conditions. The factory owner assures him that the kids benefit immensely, because they make money for their families and receive some education. The interviewer doesn't share the factory owners same enthusiasm for child labor, but he doesn't publically express his discontent. You really get the feel for how people justified child labor back in those days.

Document 6

In this reading from the Parliamentary Papers, the wise factory owner explains his reasons for working his employees less hours. The factory owner states that there is no reason to work the employees so many hours because after about 11 hours, they stop being efficient or productive or the quality suffers. He states, "there is more bad work made the last one or two hours of the day than the whole of the first nine or ten hours." That was his justification for reducing their work hours "without the slightest fear of suffering loss" of profits. This perspective is unique because it is a capitalist reason for reducing work hours, as opposed to a communist perspective which would argue that all bosses are bad because they enforce the inhuman capitalist system. This document, on the contrary, shows that there were capitalist worker reformers in addition to the communists.

Document 7

This document is from the Communist Manifesto by Karl Marx and Freidrich Engels- the most important document written during the industrial

revolution. The beginning of the document explains that the history of the world has always been between two classes of society- rich and poor, landlords and peasants. The only way for workers to get their fair share of the profits is to overthrow the bosses in a violent revolution that leads to a world controlled by the workers. The goals of communism were to unite all workers from around the world. The final words of the document were a call to action, "The proleterians have nothing to lose but their chains. They have a world to win. Working men of all countries, unite!" Those are the words that have influenced so many communist and socialist movements since they were written in 1848. It was written right at the peak of industrialization as a result of the mistreatment of workers like the others you read about in this DBQ. This document is in direct opposition to the wise factory owner document (6) because it says that there is no such thing as a good capitalist. To them, a good capitalist (factory owner) is like a good slave owner- meaningless. They're both horrible because the factory systems and slavery were inherently evil; there was nothing any good slave or factory owner could do to make it right since the workers weren't in control of their own labor in either system. He refers to wage slavery as worse than chattel slavery, because the workers were paid by the hours and it didn't matter if they had food, clothing or shelter. However, at least in slavery, the slave had food, and shelter to stay alive and work the next day. Some communists believed that slaves were more valuable than hourly wage workers or " wage-slaves", because slaves were taken care of, but workers were discarded after their work hours were completed.

SECTION II, PART B: LONG ESSAY QUESTION (EXPLAINED)

Question 2

It is important to know some key facts and details for your essay like...

(IDEAS & FACTS TO MENTION IN YOUR ESSAY)

Before the revolution, there was a civil war, Nationalists v Communists

Sun Yet Sen and then Chiang Kai Shek were the nationalist leaders.

Mao Ze Dong was the Communist leader.

During WWII, the forces united versus their common enemy, Japan.

In 1949, the Communists came to power in China.

Chinese communism was not traditional.

Chinese communism was based on a peasant revolution, not workers.

Mao wanted to apply the concepts of Marx to China.

The Cultural Revolution spread across China.

Revolutionaries promoted Communist propaganda all over.

The Great Leap Forward was a failed program that led to famine.

Communism in China is not viewed by outsiders as a success.

The peasants remained mostly poor until globalization, after 1973.

In 1973, Deng Xiaoping opened up China to Western businesses.

Communism w a mix of capitalism continues today in China.

SECTION II, PART B: LONG ESSAY QUESTION (EXPLAINED)

Question 3

It is important to keep in mind that India was the "crown jewel" of the British empire. When India gained independence, many other nations

thought that their dreams of liberation would soon come true. Were they right? Let's look at a few key facts and concepts.

(IDEAS & FACTS TO MENTION IN YOUR ESSAY)

Before 1947, Europeans held most of their colonial possessions.

After 1947, India gained independence.

India and Pakistan were both formed in 1947.

After India, many other nations gained independence.

India gained independence by using non-violent tactics.

Algeria gained independence using violent resistance.

Africa gained most of its independence in the 1960's.

Ghana gained independence in the 1950's.

Many nations gained independence after 1947.

Most of the colonies didn't get their freedom until 20 years later.

Vietnam was decolonized in the 1950's.

After the French left Vietnam, the Americans invaded the country.

In 1997, Hong Kong went back to China.

The Treaty of Nanjing in 1847 gave Britain control of Hong Kong.